WK 5 Head Trauma and TBI

Réussis tes devoirs et examens dès maintenant avec Quizwiz!

Specific traumatic brain injuries (TBI): Skull fractures: Basilar skull fractures: Emergency interventions

(1) Semi-Fowler's position unless hemodynamically unstable. Avoid nose blowing, straining, sneezing, sucking through straws and/or using an incentive spirometry device to limit meningitis risk (Sinus Precautions). (2) Usual cause for persistent bleeding from ear is canal laceration (3) CSF leak may be helping to blunt a rise in ICP and limit brain damage. Do not try to stop it. Place nothing in the nose or ear. Collect drainage from nose on a rolled 4 X 4 (moustache dressing) taped over the upper lip. Place a loose dressing over the ear to collect drainage. If mixed with blood, look for the characteristic "halo" sign as blood cells are heavier and will stay in the middle while CSF wicks out to the perimeter causing a larger straw-colored ring. "This test uses the principle of chromatography: different components of a fluid mixture will separate as they travel through a material. Although the value of this sign has been debated, an experiment showed that the sign was consistently visible when CSF concentrations were 30%-90% when mixed with blood. However, the sign was not specific to CSF: mixtures of blood with saline, tears or rhinorrhea also produced halos; filter paper, paper towel, coffee filters and linen all showed a ring". (4) Prophylactic antibiotics are controversial. Eighty percent of CSF leaks self-seal. If it persists longer than 7-14 days, may need dura repair. Prophylaxis with Pneumovax may be indicated.

Specific traumatic brain injuries (TBI): Focal injuries: Blunt Cerebrovascular injury

1-3% of patients admitted following blunt trauma will develop cerebrovascular injury of the carotid and vertebral systems. a. Suggested screening criteria include the following (1) Level I: No level I recommendations can be made. (2) Level II: (a) Patients presenting with any neurologic abnormality that is unexplained by a diagnosed injury should be evaluated for BCVI. (b) Blunt trauma patients presenting with epistaxis from a suspected arterial source after trauma should be evaluated for BCVI. (3) Level III: (a) Asymptomatic patients with significant blunt head trauma as defined below are at significantly increased risk for BCVI and screening should be considered. Risk factors are as follows: (i) GCS score ≤8; (ii) Petrous bone fracture; (iii) Diffuse axonal injury (iv) C-spine fracture particularly with fracture of C1-C3 and fracture through foramen transversarium (v) Cervical spine fracture with subluxation or rotational component; (vi) Le Fort II or III facial fractures b. Pediatric trauma patients should be evaluated using the same criteria as the adult population. c. Diagnostic Imaging: CTA, Angiography, MRA d. Treatment: Endovascular coiling / stenting; Anticoagulation / antiplatelet therapy if no contraindication. (often, only aspirin can be given initially due to multitrauma)

Lab profiles

A. H&H or CBC B. Glucose (injured brain is hypermetabolic and glucose intolerant; levels increase in intracranial hemorrhage; decrease in secondary ischemia. Glucose >200 = poor outcome) C. Serum lactate D. Electrolytes E. Drug/tox screen F. ABG, SpO2, AVdO2, SjO2. G. Creatinine, blood urea nitrogen (BUN) H. International normalized ratio (INR) I. Prothrombin time (PT) and partial thromboplastin time (PTT) J. Urine electrolytes, urea and glycerol q. 6 hours. Monitor Na levels for SIADH. K. Serum osmolarity: diagnoses injury to hypothalamus which may result in diabetes insipidus as indicated by serum osmolarity > 295 mOsm/kg, or syndrome of inappropriate ADH secretion (SIADH) with serum osmolarity less than 280 mOsm/kg.

Common etiologies

1. Falls have surpassed motor vehicle crashes (MVCs) as the most common cause of closed head injury. Prior TBIs accounted for 46% of the fatal falls. TBI-related mortality begins to rise at age 30 and continues to increase each decade Falls are seen more frequently in children and the elderly. Other etiologies: MVCs, use of firearms, intentional battery, water or recreational or sport-related injuries, and struck by or fallen against. 2. Factors that increase the risk of sustaining head trauma a. Alcohol ingestion; use of mild-altering drugs b. Incorrect use or nonuse of restraint systems; nonuse of helmets c. Participating in team sports without protective equipment d. Aging; use of anti-coagulation medication

Adjuncts to primary survey

1. Place urinary catheter as ordered; carefully monitor and record intake and output. 2. Insert an NG or OG tube depending on the nature of the trauma to prevent gastric distention and to decrease the risk of vomiting and aspiration.

Alteration in thermoregulation

1. Prophylactic hypothermia is NOT recommended. Hypothermia is well recognized to preserve cells and tissue in the face of metabolic challenge. The current standard for neuroprotection after cardiac arrest from acute coronary syndromes does NOT translate to central nervous system trauma. Though hypothermia is well known for its ability to reduce intracranial pressure, it bears risks, including coagulopathy and immunosuppression. Profound hypothermia bears the additional risk of cardiac dysrhythmia and death. 2. Adjust room temperature as needed 3. Adjust patient covering as needed 4. Monitor for fluctuations in temp and alert physician if either occurs 5. Administer hypothalamic depressants as ordered 6. Deploy invasive technology (ie, CoolGuard®) for controlled euthermia if noninvasive fever control measures are ineffective

Specific traumatic brain injuries (TBI): Diffuse injuries

>80% of all TBIs; 50% of all admits; 1/3 of all TBI deaths 1. There is no macroscopic brain injury that can be seen immediately on skull film or CT scan. However, there is wide-spread microscopic disruption of both structure and function due to shearing, stretching, or tearing forces applied to nerve fibers. Coma is due to direct damage of brainstem or cortex, but no brainstem compression.

Advanced Monitoring Strategies (Infrequently used outside of research ICUs)

1. The goal of the medical management of severe traumatic brain injury (TBI) is to ensure that nutrient delivery to the brain is optimized through the period of abnormal physiology and brain swelling that follows injury. The only way to be assured that this is being achieved to the greatest extent possible is to measure brain metabolites which provide reassurance that the needs of oxidative metabolism are being met. 2. In recent decades, invasive monitors have been developed that monitor brain pressure, oxygenation (PbrO2), blood flow and cerebral metabolism on a continuous or nearly continuous basis. Current placement strategies are not precise and there is insufficient understanding of how specific brain regions and distance from focal lesions affect their measurements. 3. Substantial gaps in our knowledge currently exist regarding how the data provided by advanced cerebral monitors should be used. Uncertainty remains as to what and/or if a specific threshold best characterizes the relationship with outcome. It is critical to consider these limitations and knowledge gaps when examining the literature support use of these technologies for patient care. 4. Transcranial Doppler -Used to detect vascular injury and has been replaced by the CTA in most settings. Common carotid branches into the external carotid that perfuses the face and the internal carotid that perfuses scalp and brain. The velocity of the middle carotid artery (MCA)/external carotid = 1.7; >3 = vasospasm. 5. Systemic jugular venous oxygen saturation (SjVO2 or SjO2), Parenchymal probe that assesses real time O2 availability to the brain by evaluating arterial-jugular venous O2 content differences (AVdO2) (extraction ratios). SjVO2 and CBF monitoring may help to identify cerebral ischemia if hyperventilation values < 30 mmHg are necessary. a. Monitor patients with TBI, SAH in vasospasm, and post-op patients with suspicion of significant brain swelling or intracranial hemorrhage. b. Dependent upon CBF and HbO2 content c. Normal SjO2 = 65-75%; goal: keep above 55% d. SjO2 decreases with (1) decreased CBF which increases O2 extraction; (2) anemia; and (3) a decrease in arterial oxygen extraction. (4) If SjO2 is < pO2 27 - look for above e. Cerebral blood flow decreases with (1) increased ICP and decreased CPP; (2) excess hyperventilation; (3) cerebral spasm; and (4) systemic hypotension. f. BTF Recommendations (2016): (1) Level III: Jugular venous saturation of <50% may be a threshold to avoid in order to reduce mortality and improve outcomes. (2) Level III: Jugular bulb monitoring of arteriovenous oxygen content difference (AVDO2), as a source of information for management decisions, may be considered to reduce mortality and improve outcomes at 3 and 6 months post-injury. 6. Brain Tissue Oxygenation (LICOX) monitor a. Monitoring the partial pressure of brain tissue oxygen (PbtO2) helps to prevent hypoxic events and improve patient outcome. It can predict hypoxic events sooner than ICP / CPP monitoring alone. b. PbtO2 monitoring adds to the information needed to target and balance therapy in patients as they respond to changes in blood flow, decrease in energy production, alteration in cellular response, and potential ischemia. c. The LICOX oxygen/temperature probe is placed in the brain parenchyma. Tissue oxygen values of 20 mmHg or higher are thought to be an acceptable starting point. d. Nursing implications (1) Studies show a drop in PbtO2 when suctioning if preoxygenation is omitted and when changing IV tubing used to administer vasopressors. (2) Hyperventilation reduces ICP and increases CPP, but brain tissue oxygen decreases to as low as 10 mmHg due to vasoconstriction, confirming why hyperventilation in the first 24 hours after brain injury can cause hypoxia in the tissue at risk. (3) There is an alarming rapid drop in PbtO2 not revealed by other monitoring devices when a patient's head is turned - supporting the need for axial alignment.

Surgical management: Operative considerations

1. There have been variations in surgical techniques, timing, and patient populations in most of the observational studies published in the last 2 decades. b. Level II A bifrontal DC is not recommended to improve outcomes as measured by the Glasgow Outcome Scale-Extended (GOS-E) score at 6 months post-injury in severe TBI patients with diffuse injury (without mass lesions), and with ICP elevation to values >20 mm Hg for more than 15 minutes within a 1-hour period that are refractory to first-tier therapies. However, this procedure has been demonstrated to reduce ICP and to minimize days in the ICU. c. A large frontotemporoparietal DC (not less than 12 x 15 cm or 15 cm diameter) is recommended over a small frontotemporoparietal DC for reduced mortality and improved neurologic outcomes in patients with severe TBI d. NEW-Secondary DC performed for late refractory ICP elevation is recommended to improve mortality and favorable outcomes. e. NEW-Secondary DC performed for early refractory ICP elevation is not recommended to improve mortality and favorable outcomes. f. A large frontotemporoparietal DC (not less than 12 × 15 cm or 15 cm in diameter) is recommended over a small frontotemporoparietal DC for reduced mortality and improved neurological outcomes in patients with severe TBI. g. NEW-Secondary DC, performed as a treatment for either early or late refractory ICP elevation, is suggested to reduce ICP and duration of intensive care, though the relationship between these effects and favorable outcome is uncertain. 2. Craniotomy with clot evacuation is considered emergently for intracerebral hematomas causing substantial mass effect. These include: an extracererbral hematoma > 1 cm thickness or intraparenchymal hematoma >3cm diameter with associated >5mm midline shift. Anticipate the need to prepare the patient for craniotomy if present. Coagulopathies should be reversed and ICP controlled to the extent medically possible without further patient jeopardy prior to the surgical procedure to limit complications. 3. Unstable comatose patients who are taken to surgery for thoracic or abdominal injury may be candidates for diagnostic burr holes or insertion of ICP monitoring if there is a significant scalp injury or signs of herniation. 4. Temporal Burr Hole: Used as a temporizing measure in situations where immediate neurosurgical care is not available by a surgeon properly trained in the procedure. Indications: hemiplegia, dilated pupil (85% dilate on side of mass; 15% dilate contralateral to mass (Kernohan's Phenomenon)). Key points: a. Most head injured patients do not have hematomas b. A burr hole may fail to locate the hematoma c. Only a small portion of the hematoma can be evacuated through burr hole d. Procedure may produce brain damage and may not be life-saving e. Time spent performing procedure may = time to get pt to a neurosurgeon f. Should only be done with proper training AND the advice and consent of a neurosurgeon 5. In extreme cases of cerebral edema, surgeons may do a decompressive craniectomy to remove a portion of the skull to allow for brain swelling- prevent herniation and death. Pts may be fitted w/ a protective helmet until the skull defect can be reconstructed or repaired- generally occurs 6-12 wks post op.

Ongoing patient monitoring

A. A patient is considered in coma until they can obey commands and localize pain B. VS as ordered C. Mentation, orientation, responses and GCS D. Neuro signs including pupils, motor strength and sensory response to stimuli E. Report any change from previous assessments F. A patient is deteriorating if: 1. GCS drops by 2-3 points; 2. there is a unilateral pupil change; or 3. there is lateralized extremity weakness.

Sequelae of head trauma

A. Glasgow Outcome Scale 1. Good recovery 2. Moderate disability 3. Severe disability: never return to work 4. Persistent vegetative state (very costly care) - awake but unaware 5. Death B. Anticipate poor outcomes if any of these are present: 1. Hypoxia 2. Shock (hypotension) with ↑ ICP 3. Hypercarbia 4. Hyper or hypoglycemia 5. Hypercalcemia 6. Hypermagnesemia 7. Drop in cerebral blood flow 8. CPP < 50 and ischemic brain injury 9. 40% mass lesions 10. GCS that drops 2-3 points 11. Pupil asymmetry (acute with AMS) 12. Unilateral extremity weakness and drift/leg lag C. Predicted Outcomes: An international study examining >10K patients, was able to validate prognostic models for death at 14 days and death and disability at six months in patients with TBI. The two models use clinical exam only or clinical exam plus CT scan data. Findings: 1. Older age, low GCS, absent pupil reactivity, and major extracranial injury predict poor prognosis. 2. "Older age" was seen as a negative factor only after age 40. 3. "Obliteration of third ventricle or basal cisterns" on CT was associated with the worst prognosis at 14 days. 4. Clinicians can access the web based calculator to enter data and compute a prognostic score @ at www.crash2.lshtm.ac.uk/. Authors caution that clinical judgment must be used in conjunction with this mathematical model. D. Rehabilitation: Comprehensive neuropsychological rehabilitation involves combination therapies for persons with cognitive, emotional, interpersonal and motivational deficits associated with TBI and is beneficial for symptom management, community reintegration, and return to work.

Neuro diagnostic radiography

A. A normal PE, normal CT, and a normal MRI DO NOT mean a normal patient B. X-Rays: Skull films are no longer recommended by the Am College of Neuroradiology for most patients (ACR, 2021). Note Pencil lead doesn't show up on X-ray). C. C-T scan: Probably the most useful diagnostic tool in the emergent treatment period due to rapid study completion and greater availability than MRI. CT shows neurons or gray matter (not axons), hematomas, contusions, fluid-filled ventricles, mass lesions or localized injury requiring operative intervention, and associated bony structures. They do not detect diffuse injury. Scans will show if there is a shift of midline structures and reveal evidence of brain edema. Significant pathology can occur even with a presumably negative initial CT scan particularly with cerebral contusions that commonly develop hours to days after injury. Delayed epidural and subdural hematomas have also been reported thus a lower threshold for imaging exists in elders an those who are anticoagulated. ICP cannot be reliably predicted by CT alone. 1. The ACS has adopted the Canadian CT Head Rule as a guide to clarifying when CT scans of the head should be used. The rules do not apply to kids younger than 16 or patients who are anticoagulated. Canadian CT Head Recommendations: any of the base criteria for Mild Head Injury below PLUS one factor in High or Medium risk sets that follow • Witnessed loss of consciousness • Definite amnesia • Witnessed disorientation with concomitant GCS 13-15 High-risk of neurosurgical intervention Glasgow coma scale score < 15 at 2 hours after injury Suspected open or depressed skull fracture Any sign of basilar skull fracture Two or more vomiting episodes Age ≥ 65 years Medium risk for brain injury on CT if: >5minute loss of consciousness Amnesia before impact greater than 30 minutes. Dangerous mechanism (pedestrian, ejected from car, fall greater than three feet or five stairs). 2. Other criteria that suggest the need for CT a. Alcohol or other drug intoxication b. Anticoagulant use or bleeding disorder c. Patients with shunts from prior neurosurgical procedures d. Patients who return after initial evaluation following a head injury e. Combativeness f. Facial injury g. Acute pupillary inequality h. Skull films reveal IC air or shift of the pineal gland from midline i. Amnesia, particularly antegrade j. Infants and children with head injury: Clinical features can be very subtle in these populations; however, radiologic exposure is a concern. PECARN (Pediatric Emergency Care Applied Research Network) developed an evidence-based decision making algorithm for Head CT Imaging in Pediatrics Group A = <2yrs of age Group B = >2yrs of age 3. NON-CONTRAST in most cases. If basal cistern is absent, anticipate 88% mortality in 24 hours 4. Need iron in blood to see it on CT. There are some isodense subdurals. If there is anemia from trauma with a ↓ Hct, may not see the clot on CT. D. Cerebral angiography: Defines vascular injury (preferentially replaced with initial MR / MRA for speed. Commonly seen with penetrating and/or neck trauma, basilar skull fractures. Transfemoral approach preferred. E. CT Angiography (CTA): Screening tool for vascular lesions including vertebral / carotid dissection. Recommended with high cervical, basilar skull and condyle fractures. Contrast is given through peripheral IV during CT scanning. F. Magnetic resonance imaging (MRI)/MR angiography (MRA): Uses computer technology to convert radiofrequency signals into shades of black, gray, and white. The unit produces a three dimensional plot of proton densities. Forty-one percent of TBI patients with a normal CT will have an abnormal MRI. Increased sensitivity reveals small or subtle lesions. Paramagnetic agents (gadolinium) are used to enhance the image. MRI is more useful in subacute and chronic phases of head injury. 1. Advantages: Does not use ionizing radiation, can better discriminate between similar tissues, is better at defining cerebral edema, not complicated by bone artifacts, provides delineation of structures in three planes (axial, sagittal, and coronal). In acute trauma, MRI may be useful in assessing the extent of parenchymal injury, the exact location of extra-axial blood collections (i.e., subdural, epidural, and subarachnoid hemorrhage), the integrity of vascular structures using MR angiography, and potentially to determine the metabolic impact of injury using MR spectroscopy. 2. Disadvantages during the acute phase: cost, limited availability, speed of scanning, potential for motion artifact, limitations on the use of life support equipment in the strong magnetic field, insensitivity to bone involvement G. SPEC scan: Single Photon Emission Scan: measures metabolism of the brain. Useful during Rehab phase. They can see if the brain is "idling" and try to determine if it is a biochemical or electrical event. H. PET scan: similar to SPEC. Nuclear med based test. Used for prognostic / targeted functional therapy during the rehab phase of TBI care. I. Other studies 1. Lumbar puncture: used when post-traumatic meningitis is suspected 2. EEG 3. Evoked potentials; visual, brain stem auditory, and somatosensory 4. Xenon blood flow studies

Secondary assessment

A. Chief complaint and SAMPLE history: What are the patient's current symptoms? 1. Nursing diagnosis: Pain. Headache may be due to cerebral edema, vascular dilation, traction on bridging veins, or stretching of the arteries at the base of the brain. a. Ask patient about the onset, provocation/palliation efforts, quality (throbbing, constant/intermittent etc.) region/radiation/recurrence, severity (0-10), and time (how long has this lasted?) b. Ask about associated S&S such as visual disturbances, nausea, vomiting, sinus congestion, photophobia, etc. c. If patient has a known hematoma, contusion, cerebral edema, aneurysm, or AV malformation, worsening headache may signal an increase in the size of the lesion, additional bleeding, or brain tissue swelling. Notify a physician immediately. 2. SAMPLE history a. If conscious, ask if they lost consciousness (or remember waking up) b. Did the patient vomit? c. Allergies d. Has the patient ingested any drugs or alcohol? e. Medications: "Patients on oral antithrombotics (OATs), both warfarin and antiplatelet agents, are at increased risk of intracranial hemorrhage (ICH) after trauma. Similar increases in mortality have been seen in trauma patients on antiplatelet agents, although the data are less robust, with some studies demonstrating increased mortality and others failing to do so. Many authors, including the ACS Committee on Trauma recommend a very aggressive approach to anticoagulated patients: immediate triage, rapid physician evaluation, head CT with immediate reading; and anticoagulant reversal (see table) There is evidence to suggest that rapid recognition and reversal of anticoagulation in patients with ICH can decrease progression of hemorrhage and improve outcomes. The evidence supporting platelet transfusion in patients on antiplatelet agents is less robust, but transfusion may reduce mortality in patients with ICH". The CRASH-3 study suggests a mortality benefit following moderate head injury if tranexamic acid (TXA) is administered within three hours of injury (Rarajee, UpToDate, 2021). Admit to ICU or take to surgery if required. If CT is negative, admit for 24 hours of observation. f. Significant underlying illnesses g. Last oral intake h. Ask about mechanism of injury and events surrounding the trauma

Patient destination and trauma team composition for optimal outcomes

A. Need 5 R's for optimal outcomes 1. Right patient, to the 2. Right hospital, in the 3. Right amount of time, to be cared for by the 4. Right physician (neurosurgeon) and trauma team and receive 5. good Rehabilitation. Right hospital note: Patients with severe traumatic brain injury should be transported directly to a Level I or Level II trauma center that offers CT scanning, neurosurgical care, ICP monitoring, and treatment capabilities, even if this center may not be the closest hospital. Transport decisions in the field are among the most important decisions affecting outcome in patients with severe TBI. When an integrated EMS and trauma system is in place and EMS agencies transport a patient directly from the scene of the incident to an appropriate trauma center, the patient is entered into a system of care that has been shown to improve overall patient outcome. Inter-hospital transfers of these patients are known to delay the time until neurosurgical consultation and intervention occur at a time of great risk for secondary insult to the brain. B. Traumatic brain injury care team should include the following: 1. EMS providers: ground and/or air transport 2. Physicians specializing in emergency medicine, trauma, neurosurgery, neurology, radiology, and anesthesiology 3. Nurses from the ED, Neuro/Trauma ICU and/or trauma service, Neuro Med-Surg Units, OR and PACU (some would include case management, discharge planners) 4. Ancillary departments: CT, radiology, lab, pharmacy, physical medicine and rehabilitation, speech therapy and dietary 5. Social services 6. Pastoral care

Advanced cerebral monitoring and interventions (Used outside of the resuscitative phase)

A. The goal of the medical management of severe TBI is to ensure that nutrient delivery to the brain is optimized through the period of abnormal physiology and brain swelling that follows the injury. The only way to be assured that this is being achieved to the greatest extent possible is to measure brain metabolites which provide reassurance that the needs of oxidative metabolism are being met. B. Historical means of examining brain health, such as the Kety-Schmidt method, which remains a gold standard assay for cerebral blood flow and metabolism, as well as xenon-CT, which informs the former, were cumbersome. Both provide information about large brain regions, as does jugular venous O2 monitoring (SjO2). C. In recent decades, invasive monitors have been developed that monitor brain pressure, oxygenation (PbrO2), and blood flow on a continuous or nearly continuous basis. D. Microdialysis measures metabolites in the brain's extracellular fluid. (glucose, lactate, pyruvate, and glutamate) and electrocorticography determines cortical spreading depression; however, use of these last two monitoring techniques is not common outside of research settings. E. Substantial gaps in our knowledge currently exist regarding how the data provided by advanced cerebral monitors should be used. These gaps are substantially greater for some such technologies than others. Studies published to date have attempted to explore putative thresholds of prognostic significance; however, uncertainty remains as to the precise thresholds that should be employed, and if the notion of a threshold best characterizes the relationship with outcome. F. For regional monitors, there is insufficient understanding of how specific brain regions and distance from focal lesions affect measurements. Moreover, placement of these monitors with stereotactic precision is not currently feasible for these devices. It is critical to consider these limitations and knowledge gaps when examining the literature supporting use of these technologies for patient care. G. Advanced cerebral monitoring techniques for blood flow and oxygen include: transcranial Doppler (TCD)/duplex sonography, differences between arterial and arterio-jugular venous oxygen (AVDO2), and measurements of local tissue oxygen. Arterio-jugular AVDO2 globally measures cerebral oxygen extraction. However, the measured AVDO2 can potentially differ from the other unmeasured hemisphere in TBI patients. H. Tissue monitors are placed in the cerebral cortex and directly measure tissue oxygen in the immediate area. The relationship between brain tissue oxygen, oxygen delivery, and diffusion of dissolved oxygen across the blood brain barrier is not simple, and most studies using tissue oxygen monitors treat initial desaturation episodes with 100% inspired oxygen rather than a transfusion of red blood cells or vasopressor administration to improve cerebral perfusion pressure (CPP). I. Theoretically, use of advanced monitoring in tandem with intracranial pressure (ICP) and CPP monitoring adds to the assessment of brain metabolic needs and the effects of therapies to meet them. However, all techniques have limitations and potential risks. J. Level III: Jugular venous saturation of <50% may be a threshold to avoid in order to reduce mortality and improve outcomes. K. Level III: Jugular bulb monitoring of arteriovenous oxygen content difference (AVDO2), as a source of information for management decisions, may be considered to reduce mortality and improve outcomes at 3 and 6 months post-injury.

Pathophysiology of secondary brain injury

A. The initial trauma sets in motion a series of molecular events which activate endogenous substances, i.e., oxygen free radicals, monoamines, neuropeptides, arachidonic acid metabolites, and alters calcium metabolism. B. Oxygen free radicals: Superoxides and lactic acid are released as a result of mitochondrial dysfunction in the absence of obvious ischemia. These oxygen radicals cause lipid peroxidation of polyunsaturated fatty acids disrupting cell membranes and possibly resulting in the breakdown of the blood-brain barrier and progressive axonal degeneration. C. Either through overt damage or by some other mechanism, voltage gates burst open releasing bradykinin, kallikrein, excitatory amino acids (particularly glutamate which is the most excitatory neurotransmitter), and arachidonic acid and its metabolites. D. Current research is exploring the causal relationship between these substances and the development of brain edema and secondary brain damage. Glutamate is stored inside of cells or is shuttled discretely between them. The temporal lobe is filled with glutamate. When trauma occurs, this major neurotransmitter spills freely. E. There are mechanisms responsible for glutamate uptake by neurons, but these mechanisms quickly become overwhelmed when there is an excess of glutamate. ATP is needed to pump it across the membranes. F. When the flood of glutamate pours into synaptic clefts, it triggers the opening of key ion channels in the neuron that receives the glutamate signal and over stimulates the neurotransmitter's receptors with a subsequent rush of ions across the cell membrane wall, particularly passage of calcium through N-Methyl-D-Aspartate [NMDA]-receptor mediated channels. Potassium is flushed out of the cell. Calcium is considered essential in normal neuronal activity and is normally activated in small amounts. It reshapes parts of the cell wall membrane under controlled conditions. The drop in energy availability after injury triggers uncontrolled calcium activity late in the cascade of events. G. One of the first effects of this Ca influx is that glycolysis is stepped up to provide more energy to pump ions across the cell membrane. The Ca blockers used in CV disease are not very effective in blocking the Ca channels in the brain. Brain ion channels have very unique properties. H. Protein synthesis that is needed for normal membrane permeability is slowed. The inhibition of protein synthesis (especially in the temporal lobe) may be one cause of post- concussive amnesia. I. Glycolysis may initially protect the cell by helping to correct the ion imbalance, but it steadily increases the amount of lactic acid in the cell through anaerobic metabolism. Studies have shown that immediately following injury, glucose metabolism increases, but that this trend is followed by a prolonged decrease that lasts several days. Treating animals with excitatory amino acid antagonists greatly decreased the brain's demand for glucose. J. The resulting acidosis leads to a breakdown of the cell membrane with fluid influx and bloating of the cell (cytotoxic edema). This causes self-destruction (apoptosis) and eventual death. A similar progression can be charted in both contusion and stroke, although the timing and topography of the events are different. Within seconds of trauma, the release of fatty acids from the cell membrane is initiated. The entry of calcium ions into the cell is followed by lipolysis, then proteolysis, and finally protein phosphorylation, which may ultimately be what kills the cell. K. Summary: Secondary injury causes breakdown of the blood brain barrier, disruption of cerebral autoregulation, accumulation of toxic extracellular levels of excitatory amino acids and free radicals, and initiates cellular inflammatory responses and regional hyperthermia. It diminishes the effectiveness of autoregulatory and compensatory mechanisms of the brain, thus compromising perfusion and producing ischemia.

Morbidity

Brain injury can result in memory loss, rapid mood swings, fatigue, intellectual deficits, mental rigidity, personality changes, and physical disabilities. The terms mild moderate and severe traumatic brain injury are used to describe the level of initial injury in relation to the neurological severity caused to the brain. "The categorization of traumatic brain injury reflects a continuum".There may be no correlation between the initial Glasgow Coma Score and the initial level of brain injury and a person's short or long-term recovery or functional abilities.

Hyperventilation results in hypocapneic cerebral vasoconstriction and brain ischemia

BTF Guidelines 2016 recommend "recognizing the potential need for hyperventilation as a temporizing measure" (p 63). It is only indicated in impending cerebral herniation for <20 minutes and not used for ventilator settings. Ventilation parameters to maintain a PCO2 of approximately 35 mmHg. Reserve hyperventilation acutely in patients with severe brain injury to those with acute neurologic deterioration or signs of herniation. Prolonged hyperventilation with PCO2 < 25 mmHg is not recommended. d. There is an improved survival rate for intubated patients with pCO2 values ranging between 30 and 49 mmHg with a rapid decrease in survival for results less than 30 or over 49 mmHg. This underscores the potential dangers of intubation associated with positive pressure ventilation and hyperventilation. The use of ventilators or capnometry-guided ventilations for patients with severe TBI is important. e. If intubated prior to arrival, check pCO2/EtCO2, bilateral breath sounds and tube patency. f. If not intubated and procedure is indicated: Prepare equipment for patients with a GCS ≤ 8 as many have a pO2 < 60 torr and are hypercarbic. (1) If awake or responsive to pain and/or gag reflex present: Drug-assisted intubation (RSI) with in-line stabilization. Premedicate the hypopharynx with a topical anesthetic. Some physicians still give lidocaine as a premedication prior to RSI. Lidocaine is a sodium channel blocker and acts like a "surge protector" for the brain, blunting the sympathetic surge that may occur during the intubation process in a response patient. Give sedatives and paralytics as prescribed and authorized by your scope of practice. (2) If unresponsive and/or apneic: Orotracheal intubation with in-line stabilization. Nasotracheal intubation is avoided in the presence of midface / anterior basilar skull fractures due to the risk of concomitant cribriform plate fracture and cranial vault penetration. g. If intubation attempts fail and the patient cannot be adequately ventilated with a BVM, anticipate the need for an alternate or surgical airway. h. Gold standard in children may be NO intubation. Insert an oral airway and attempt ventilations with a BVM.

Review of systems

Exam of head should include inspection for deformities, asymmetry, contusions, abrasions, puncture wounds, bruising, lacerations, swelling/edema, bleeding, and drainage of CSF from eyes, ears (otorrhea), nose (rhinorrhea), or mouth. Inspect eyes, nose, mouth, and ears for signs of injury.

Focused neuro exam

Extent of exam depends on the patient's level of consciousness and acuity. If awake, alert and cooperative, can perform detailed assessment. If comatose, the nursing exam is usually limited to GCS, pupillary check, and pain responses.

Moderate brain injury

GCS is 9-12. These injuries result in a loss of consciousness usually lasting minutes to a few hours. It is followed by a few days or weeks of confusion, and may be accompanied by brain contusions or hematomas. Although people usually have physical, cognitive and psychosocial or behavioral impairments that may last several months, treatment will allow 60% to recover fully; 25% will have moderate disability; 7-10% will die or have persistent vegetative state.

Severe brain injury

GCS of 8 or less. Severe injury almost always results in prolonged unconsciousness or coma lasting days, weeks, or longer. Complications include brain contusions, hematomas, or damage to the nerve fibers and some may have suffered from anoxia. It is sometimes possible to make significant improvements in the first year after injury that can continue to improve slowly for many years with excellent rehabilitation. Only 25-33% will have a positive outcome; 33% will die and the rest have moderate - severe disability. These patients will be left with some permanent physical, behavioral, and/or cognitive impairment. Severe brain injury is further categorized into subgroups with separate features: a. Coma b. Vegetative state c. Persistent vegetative state (PVS) d. Minimally responsive state e. Akinetic mutism f. Locked-in syndrome

Subacute SDH

Interval from Injury: 3 days - 3 wks Clinical Presentation: Similar to acute (often see mixed acute & subacute with repeat falls in elders) CT Blood Appearance: Isodense (gray)

Chronic SDH

Interval from Injury: 3 wks - months Clinical Presentation: Progressive headache, Slow cerebration, Confusion, Drowsiness, Possible seizure, Papilledema, Balance issues and Contralateral hemiparesis. Hematoma may reaccumulate or calcify. CT Blood Appearance: Hypodense (black)

Acute SDH

Interval from Injury: <72 hours Clinical Presentation: Headache, drowsiness, agitation, slow cerebration, confusion. If brain is shifting: ipsilateral dilated and fixed pupil, contralateral hemiparesis. CT Blood Appearance: Hyperdense (white)

Primary (direct) injury

Mechanical injury that occurs at the moment of energy transfer. The impact or forces may cause bony deformity and disruptions to axons, cell bodies, and the integrity of cell membranes resulting in disintegration of cell structure and function and eventually cell death. Pressure waves travel across the brain and dissipate causing physical transection, shearing, bruising, bleeding, or damage of cranial contents that cannot be reversed. Disrupted blood flow to the injured area may cause ischemia and compromise of the blood/brain barrier or death of neurons. Irritation of nervous system tissue may create electrical instability. Treatment is prevention.

Focused neuro exam: Level of consciousness

Most sensitive indicator of neurological status. In a conscious patient, altered mental status (AMS) is the first sign of deterioration. To provide consistency, describe patient's response in specific behavioral terms. a. Arousal (1) Alert: Awake, responds immediately to commands (2) Lethargic: Response to commands may be incomplete or slow. Needs stimuli, but obeys. Returns to sleep. (3) Stuporous: Decreased awareness. Does not obey commands. Spontaneous or purposeful movements may be noted. (4) Comatose: Unable to arouse. Abnormal flexion or extension to stimuli. May be flaccid b. Awareness: Orientation to person (self and/or loved ones), place, time, and situation. Determine if changes are new or if they pre-date the trauma (e.g., dementia).

Focused neuro exam: Reflex exam

Not usually within a TNS scope of practice to perform; awareness of response is important to understanding the degree of deficit a. DTRs, anal wink b. Babinski: upper motor neuron (UMN) lesion c. Brainstem (1) Doll's eyes (oculocephalic reflex): Done only after C-spine is cleared in a comatose patient (a) Lift both eyelids. Rapidly turn head from midline to one side. (b) Normally, the eyes continue to look upward at the ceiling, so they appear to move in a direction opposite to the way the head is turned, i.e., when head is rapidly turned to right, eyes appear to move left. (c) Record as normal response, abnormal response (dysconjugate or asymmetrical eye movement), or pathologic response (no movement of either eye so they move in the direction the head is turned - like eyes painted on a doll). (2) Calorics; oculovestibular reflex (a) Performed when the oculocephalic test is equivocal or contraindicated (b) Verify integrity of tympanic membrane (c) Irrigate tympanic membrane with ice water (d) Normal response: slow deviation of eyes toward side of the stimulus followed by rapid override by the cortical centers to direct eyes back toward the midline. Creates impression of nystagmus away from cold. (e) Abnormal response: eye does not move (3) Gag (a) Assessed by stimulating the back of the pharynx ( tugging on the ETT or oral suctioning can produce this effect in the intubated patient) (b) Abnormal response: absence of retching or gagging (4) Corneal (a) Assessed by touching the cornea with a wisp of cotton. (b) Abnormal response: absent/no rapid closing of the eyelid.

D = Disability

Mini-neuro exam: GCS, pupils, gross motor function, and possible glucose check. Repeat assessments are crucial to monitor for presence of increased ICP. 1. Glasgow coma score a. The GCS was published in 1974 and most recently revised in 1998 by Drs. Graham Teasdale and Bryan Jennett to serve as a rapid, objective, quantifiable and reproducible tool to assess the depth and duration of impaired consciousness and coma at the bedside 24-hours after brain surgery. Teasdale and Jennett were affiliated with the University of Glasgow in Scotland, thus the city's name was incorporated into the title of the scale. b. It was not originally conceived to evaluate trauma patients. However, research has shown that severely traumatized patients in a shock-like state will experience deterioration in the GCS as it really measures brain function rather than injury, and function is affected by shock. c. Scale is used to (1) decide whether injury severity is sufficient to require or justify certain types of treatment; (2) compare different series of injuries; and (3) predict the degree of the ultimate recovery to be expected. (4) Examples (a) A single GCS measurement does not predict outcome, however, a decrease of 2 or more points with an initial GCS of 9 or lower suggests serious injury. A GCS of 3-5, as well as lack of improvement or deterioration of GCS score by 2 points or more from the field to the ED have at least a 70% positive predictive value for a poor outcome. (b) Trauma patients presenting with a GCS of 3 as well as fixed and dilated pupils in the absence of paralysis, sedation, substance abuse or the use of atropine have no reasonable chance of functional recovery. d. Scoring systems that incorporate GCS into their calculations: (1) Revised Trauma Score (RTS) (2) Trauma and Injury Severity Score (TRISS) (3) Severity Characteristic of Trauma score (4) Acute Physiology and Chronic Health Evaluation (APACHE) II and III scores (5) Circulation, Respiration, Abdomen, Motor, Speech Scale (6) Simplified Acute Physiology Score II e. Assess GCS through interaction with the patient (i.e., by giving verbal directions or applying pressure/painful stimulus to patients unable to follow commands). (1) Ideally, GCS scoring should occur after a clear airway is established, hypoxemia and hypotension have been corrected, the patient has been resuscitated and before administration of sedative or paralytic agents or after these drugs have been metabolized. (2) Components: Unconsciousness can be simplistically defined as failure to respond appropriately to environmental stimuli. Coma is defined as a state of unconsciousness from which an individual cannot be awakened, in which they respond minimally or not at all to stimuli, and initiate no voluntary activities. The GCS is the sum of three independent coded values that measure a patient's best eye opening, verbal, and motor responses either spontaneously or in response to verbal or pressure/painful stimuli. Always report their BEST response, even if different on one side from the other.

Specific traumatic brain injuries (TBI): Focal injuries

Specific, grossly observable lesions, e.g., structural or expanding mass lesions with local brain damage. Brain shifting causes coma from brain stem compression. They cause 50% of all admits and 66% of all deaths.

GCS Strengths and Limitations

Strengths: ■ Simplicity ■ Provides a common language to report neurologic findings based on bedside observations ■ Ability to trend over time Limitations: ■ Variation in inter-rater reliability ■ Inconsistent use by caregivers ■ Decreased BP (if SBP < 80, can't assess GCS with respect to possible outcomes) ■ Hypoxia, hypothermia, hypoglycemia ■ Sedating or paralyzing drugs ■ Intubated patients ■ Patients with facial/ocular trauma ■ Hearing impairments ■ Language barrier ■ Children younger than 3 years ■ Alcohol/drug intoxication ■ Language barrier or immature language skills ■ Relationship to mortality differs considerably between blunt and penetrating head trauma

Anti-cerebral edema measures

The BTF is universal in its belief that hyperosmolar agents are useful in the care of patients with severe TBI. However, the literature does not currently support recommendations that meet the strict criteria for contemporary evidenced-based medicine approaches for guideline development.

Secondary Assessment: Vital signs

The vital signs may provide very valuable information about the patient's underlying injuries. Obtain a full set of manual vital signs before hooking the patient up to automated devices. Repeat at least every 15 minutes while unstable or as indicated by local protocols. 1. Respiratory rate, patterns, and depth. Provides more clues as to the location of pathology than any other vital sign. Be particularly alert for sudden apnea and be prepared to assist ventilations. Look for diaphragmatic breathing, an indication of intercostal muscle paralysis. (Many of these abnormal patterns are masked secondary to assisted and mechanical ventilation of severe TBI patients.) a. Eupnea: Normal pattern of ventilations. b. Bradypnea: Decreased rate. Respiratory rate may slow initially after an acute increase in ICP. c. Cheyne-Stokes: Crescendo/decrescendo respirations (waxing and waning depth and rate) with periods of apnea up to 20 seconds; seen with increased ICP. Due to increased sensitivity to CO2 that results in a change in depth and rate and decreased stimulation from respiratory centers that result in apnea. Lesions are most often located bilaterally deep within the cerebral hemispheres, diencephalon (thalamus and/or hypothalamus) or basal ganglia. d. Central neurogenic hyperventilation: Regular, sustained, increased RR and depth with forced inspiration and expiration. Thought to be due to release of reflex mechanisms for respiratory control in the lower brainstem and results in decreased CO2 and alkaline pH. Giving O2 does not change pattern. Lesion location unclear, often in the midbrain and upper pons. e. Apneustic: A pause of 2-3 seconds noted after a full or prolonged gasping inspiration followed by an inefficient, brief expiration. May alternate with an expiratory pause. Lesion located in the lower pons, usually due to a basilar artery occlusion. f. Cluster: Clusters of slow irregular breaths with periods of apnea at irregular intervals (gasping breathing has features similar to cluster breathing). Lesion in the lower pons or upper medulla. g. Ataxic (Biot's) breathing: Complete irregular, unpredictable pattern with deep and shallow random breaths and pauses. Lesion in the medulla. 2. Pulse: Count rate, evaluate rhythmicity, quality, location; compare equality. 3. BP a. The traditional definition of hypotension has been a SBP <90 mm Hg, and this was the target recommended in the first iterations of the BTF guidelines. The literature now supports a higher level that may vary by age. The interrelationship between SBP, mean arterial pressure (MAP), and cerebral perfusion pressure (CPP) should be kept in mind as one considers threshold recommendations in these guidelines. b. Level III • Maintaining SBP at ≥100 mm Hg for patients 50 to 69 years old or at ≥110 mm Hg or above for patients 15 to 49 or ≥.70 years old may be considered to decrease mortality and improve outcomes. c. Blood pressure should be measured using the most accurate system available under the circumstances. It should be obtained as often as possible and, if possible, continuously by trained medical personnel. d. Once ICP monitoring has been established, manipulation of BP should be guided by CPP management. e. In children, the following SBPs have been linked to poor outcomes: (1) 0-1 years: 65 mmHg (2) 1-5 years: 70-75 mmHg (3) 5-12 years: 75-80 mmHg (4) 12-16 years: 80-90 mmHg f. Compensatory alterations in vital signs: Cushing's triad (1) ↑ SBP, widened pulse pressure (2) ↓ P (3) ↓ RR g. Decompensatory alterations (1) Hypotension (2) Tachycardia (grave sign) (3) ECG changes: (a) Q wave with ST depression (b) Prolonged QT interval (c) Dysrhythmias: atrial: PACs, biphasic Ps, A-Fib 4. Evaluate core temperature for hyper or hypothermia. Goal is euthermia. Note: The anterior hypothalamus keeps us from getting too hot (radiator). It sits right next to the pituitary stalk and is directly stimulated at 42° C. The posterior hypothalamus is the heater. It sets the hypothalamic set point and raises the body temp. The patient will change temps without sweating or gooseflesh. Each degree increase in temp = 6% ↑ in CBF.

Focused neuro exam: Mental status exam

a. Assess for short-term memory (amnesia); shortened attention span; difficulty following simple/complex commands. b. Determine if affect and behavior reveal restlessness, agitation, irritability or combativeness. Evaluate cognition by determining if the patient's responses to questions are appropriate or inappropriate. c. Ask a conscious patient if they are experiencing audio or visual hallucinations. d. Evaluate cognition and their ability to process information: Cognitive, vestibular, and anxiety/mood deficits were the most prevalent symptoms in a Brain Trauma Foundation study. Patients may need cognitive retraining. Determine if responses to questions are appropriate or inappropriate. Anticipate slowing of thought processing speed. 3. Nursing diagnosis: Alteration in cognitive function a. Attempt to orient patient to person, place, time and situation. b. Restrain patient only as necessary to provide safety.

Impaired autoregulation in TBI

a. Autoregulation is often compromised in a patient with TBI. There is a flow/metabolism uncoupling. Stimulation of the brain (increased metabolic demand) does not increase cerebral blood flow. b. If autoregulation is not intact, there is dependency on SBP to prevent cerebral ischemia, which has been ascribed to be the single most important secondary insult. If arterial pressure is ≥ 160 torr, cerebral blood volume increases. c. Increased ICP with pressure on the brain stem leads to increased MAP (Cushing's response) with no compensatory cerebral blood flow control, which further increases ICP. d. Low flow states may lead to blood brain barrier breakdown, an increase in cerebral edema, and predisposes patients to secondary brain injury from ischemia. In states of ischemia, CBF drops to 18-20 mL/100 Gm/min. At 8- 10 mL/100 Gm/min, the brain will infarct.

Specific traumatic brain injuries (TBI): Skull fractures: Basilar skull fractures

a. Definition: A fracture that involves the base of the skull b. Location: Anterior, middle or posterior fossa bones, cribriform plate, sphenoid wings, and/or petrous bones. This area is rough and ridged and has many foramina (openings) for the spinal cord (foramen magnum), cranial nerves, ear (auditory canal), and blood vessels. It forms the roof of the orbits and nasal sinuses. These spaces weaken the skull and make it vulnerable to fracture. c. Pathogenesis: Caused by blunt trauma to the head, especially to the mandible, or when the vertebral column is driven against the occipital condyles, e.g., fall on the buttocks. d. Morbidity/mortality: Risk for meningitis/encephalitis or a cerebral abscess. e. Clinical presentation: Varies by location. If patient has been supine and develops a headache when they sit up, suspect basilar skull fracture. (1) Anterior fossa (Cribriform plate, fovea ethmoidalish, sphenoid sinus) (a) Telecanthus - Medial eyelids spread out towards ear causing the bridge of the nose to appear widened and flattened. First sign ED personnel are likely to see. (b) CSF Rhinorrhea: 25% tear the dura and arachnoid, allowing CSF to leak out through the fracture site into the nasal cavity. Because CSF is high in sodium (159 mEq in CSF compared to 142 in blood), patients may c/o a salty taste in the back of their mouth. (c) Epistaxis (d) Raccoon eyes: Classic triangular bruising of the lower eye lids. Appears later. (e) May be associated with facial fractures and bleeding into orbit causing subconjunctival hemorrhages of lateral sclera giving it a blood red appearance w/o evidence of direct trauma (ocular Battle Sign). (f) Anosmia: CN I (Olfactory nerve) deficit. One third of patients will permanently lose their ability to smell, and therefore the ability to "taste" most foods. (g) Visual field deficits: CN II (Optic) (h) Pneumocephalus (i) Sinus air-fluid levels (2) Clinical presentation: Middle fossa also involves middle ear (a) CSF otorrhea: Glucose oxide reaction to glucose test tape or check for B2 transferrin (b) Hemotympanum or ear drum perforation (c) Conductive hearing deficit: 8th nerve (Vestibulocochlear) involvement (d) Dizziness; look for nystagmus (e) Facial weakness/paralysis on the affected side: Facial nerve (CN VII) runs right next to CN VIII as they exit through the base of the skull to surface structures. They are often both injured in a middle fossa fracture. (f) Battle sign: Mastoid ecchymosis presents 24-36 hrs after injury f. Diagnostic radiography: CT images of the posterior fossa with 1 to 2 mm cuts are the best way to see the fracture site. Pneumocephalus may accompany a compound basilar skull fracture. Fractures through the petrous or sphenoid bones may cause cranial nerve injuries and are best seen on high-resolution CT.

Specific traumatic brain injuries (TBI): Focal injuries: Acute subdural hematomas

a. Definition: Blood accumulation in the potential space between the inner layer of the dura and the arachnoid meningeal layers. Anatomy reminder: the subdural space extends over both sides of the tentorium, cerebellar surfaces, convexities of the brain, and the interhemispheric fissure so these bleeds will be seen crossing the suture lines and often extends along the falx and tentorium in a crescent shape. b. Incidence: 25 - 30% of head injuries. Most common traumatic mass lesion (51%-68%). Three times more common than an epidural and the worst prognosis. Two times more common in adults than children due to increased violence. c. Etiology: MVCs, falls, assaults, industrial and sports injuries. Occur from frontal or occipital impacts more often than from lateral impact. d. Populations at risk: Elderly, alcoholics, and infants as their brains are chronically dehydrated, smaller due to degeneration of cortical tissue, or smaller due to immature development. This places tension on the veins that bridge the cerebral cortex with the dural sinuses (venous outflow tracts) attached to the skull and makes them vulnerable to injury. e. Elder considerations: Minor trauma (fall from standing height), can produce a SDH.Anticoagulants add to co-morbidity. Early identification and anticoagulant reversal are key to improve outcomes. f. Pathogenesis (1) Classified as acute (within 72 hrs), subacute (3 days - 3 wks), and chronic (3 wks - months) depending on symptom onset after injury and appearance of blood on CT. (2) Bridging veins which traverse the space between the cerebral cortex and venous sinuses shear and tear. Rare arterial cause. Associated skull fractures are rare. (3) Beneath the clot, the cortex is often damaged by contusion, ruptured veins and arteries. (4) Bilateral in 8%-21% of cases (5) If less than 72 hours old, may be liquid or clotted blood (6) Associated with displacement of intracranial contents due to mass effect causing collapse of ventricular system and midline shift.

Intracranial pressure thresholds

a. Level IIB: Treating ICP >22 mm Hg is recommended because values above this level are associated with increased mortality. b. Level III: A combination of ICP values and clinical and brain CT findings may be used to make management decisions. 9. May treat ↑ ICP by draining CSF through monitoring system 10. Nursing responsibilities for patients with ventriculostomy and ICP monitors a. Observe the numeric reading and wave patterns; adjust characteristics to obtain visual reading. b. Maintain sterile site dressing. c. Adjust alarm system to unit parameters. d. Obtain frequent checks of neuro status and patency of system. Irrigate system using sterile technique to maintain patency according to unit policy. e. Turn stopcock off when turning patient to prevent over drainage of CSF. 11. Risk/complications: a. If left in place for longer than 4-5 days, infection is a risk; however routine catheter exchange for prophylaxis are not recommended b. Hemorrhage: Rare in ventriculostomy catheters. c. Malfunction or obstruction: In fluid coupled ventricular catheters, subarachnoid bolts, or subdural catheters has been reported in up to 16% of patients. d. Malpositioning

Focused neuro exam: Sensory exam

a. Superficial touch b. Superficial and deep pressure/pain c. Sensitivity to heat and cold d. Sensitivity to vibration e. Proprioception: joint position sense

Anti-cerebral edema measures: Anesthetics, analgesics, and sedatives

a. Patients with TBI may require sedation for agitation, restlessness, mechanical ventilation, or pain, to suppress metabolism in an effort to abate the "energy stress" present in injured cells, decrease ICP, facilitate effective ventilations, and provide comfort. They are used to induce a pharmacologic coma. Metabolic rate is coupled to cerebral blood flow, so a decrease in metabolism leads to a reduction in cerebral blood volume. Currently, metabolic suppression is only recommended for salvageable patients with TBI refractory to other conventional therapies such as osmotherapy, acute hyperventilation, and CSF drainage. (1) IV narcotics (morphine, fentanyl) (2) Benzodiazepines, e.g., lorazepam (Ativan) or midazolam (Versed): Benzodiazepines minimally affect ICP, cerebral oxygen demand or cerebral blood flow but can cause hypotension. (3) Diprivan (propofol) (a) Although propofol is recommended for the control of ICP, it is not recommended for improvement in mortality or 6-month outcomes. Caution is required as high-dose propofol can produce significant morbidity (b) Dose: 25-100 micrograms/kg/min IVPB (c) Advantages: Rapidly crosses over into tissues (rapid onset) within 2 to 4 minutes. Rapid offset (rapid biotransformation in the liver). Will rouse within 10-15 minutes after withdrawing the drug. Effective anticonvulsant. Not associated with a withdrawal syndrome. (d) Monitoring parameters: BP, triglycerides, creatine kinase, troponin 1, ICP, EEG (e) Adverse effects: Hypotension, hypertriglyceridemia (due to the use of fat to emulsify the drug for IV use), rhabdomyolysis, cardiac failure (propofol infusion syndrome or PIS). Because of the BP reduction, propofol may not be the best choice until hemodynamic stability is assured to prevent a drop in CPP. Conflicting data suggests a possible link with acute lung injury, so close monitoring of compliance and gas exchange is important. (f) Sedation does not provide pain management. Need to treat both if necessary. (g) FDA discourages propofol use in children for sedation or management of refractory ICP. Continuous fentanyl / midazolam drips are commonly deployed in this population. b. Barbiturates (thiopental or pentobarbital) induce an anesthesia-like state to provide cerebral protection if ICP cannot be reduced. (1) Administration of barbiturates to induce burst suppression measured by EEG as prophylaxis against the development of intracranial hypertension is not recommended. (2) High-dose barbiturate administration is recommended to control elevated ICP refractory to maximum standard medical and surgical treatment. Hemodynamic stability is essential before and during barbiturate therapy. (3) Dose pentobarbital: 10 mg/kg over 30 minutes; then 5 mg/kg every hour for three doses, then maintenance of 1-3 mg/kg/hr infusion. (4) Adverse effects: Barbiturates have a significant risk of hypotension. Monitor bowel function, acid-base status, cardiac index, ICP and EEG. (5) Barbs may be beneficial for near drowning but are contraindicated in hypotensive patients. Attempt to rule out other causes of agitation including hypoxia or electrolyte imbalance. Use extreme caution in monitoring ventilatory status. c. Neuromuscular blockade may be employed when sedation alone proves inadequate and short-acting agents should be used when possible. Unfortunately, pharmacologic relaxation has the undesirable effect of limiting the neurologic exam to the pupils and CT scan. Failure to appropriately control pain and anxiety prior to or during neuromuscular blockade may result in elevations in ICP. Therefore, its use in the absence of evidence of herniation should be limited to situations where sedation and pain relief alone are insufficient to optimize safe and efficient patient transport and resuscitation. d. Analgesics should be provided using a pain scale. The Richmond Agitation Scale and other similar scales are options to assess the need for analgesia in the nonverbal patient. 4. Maintain venous outflow from cranium a. Maintain patient's head and neck in neutral alignment to facilitate venous drainage. b. Assess airway securing and spine motion restriction devices for JV constriction. 5. Reduce environmental stimuli: Dim lights and decrease noise as much as possible 6. Evaluate patient's response to pain and consider need for analgesics. 7. Avoid increased intrathoracic or intra-abdominal pressure as these decrease venous outflow and thus, increase ICP. a. Attempt to prevent the elicitation of a Valsalva maneuver, cough, gag, vomiting or sneezing b. Avoid hip flexion and isometric muscular activity c. Minimize activities that stimulate posturing d. Recognize effect of ventilator PEEP and potential need to adjust CPP support measures (i.e., augment MAP) e. Minimize time in Trendelenburg position during central line placement f. Timely recognition and decompression of abdominal compartment syndrome 8. Avoid rapid and wide fluctuations in BP a. Beware of sudden decreases in BP - notify physician immediately b. Titrate medications slowly c. Space nursing procedures that ↑ ICP such as suctioning d. Limit environmental stimuli as indicated

Anti-cerebral edema measures: Hyperosmolar therapy: Mannitol and hypertonic saline

a. While mannitol was previously thought to reduce ICP through simple brain dehydration, both mannitol and hypertonic saline work to reduce intracranial pressure, at least in part, through reducing blood viscosity, leading to improved microcirculatory flow of blood and consequent constriction of the pial arterioles, resulting in decreased cerebral blood volume and intracranial pressure. b. Level I, II, and III: Although hyperosmolar therapy may lower intracranial pressure, there was insufficient evidence about effects on clinical outcomes to support a specific recommendation, or to support use of any specific hyperosmolar agent, for patients with severe traumatic brain injury. c. Though the 2016 BTF Guidelines no longer specify dosage and management parameters pursuant to the lack of scientific-level evidence, the utility of hyperosmolar and hypertonic solutions remain. Overview follows: Mannitol is a hypertonic simple sugar - super oxide (free radical) scavenger. It creates an osmotic gradient that pulls fluid from the intracellular and parenchymal spaces into the vascular space thereby decreasing cerebral edema, lowering blood viscosity to increase cerebral blood flow, and increasing oxygen delivery to the cells. Don't dehydrate the brain! Although mannitol can be used as a resuscitation fluid, its eventual diuretic effect is undesirable in hypotensive patients and attention needs to be paid to replacing vascular volume loss. d. Avoid hypovolemia by providing fluid replacement. If you decrease blood volume you decrease cerebral blood flow. (1) There may be a lot of cerebral swelling, but not a lot of free water. An indwelling urinary catheter is essential in these patients. (2) Maintain serum sodium levels at 145 to 150. (3) Osmoles normally = 2 X Na (143) = 286. Keep serum osmoles ≤ 320 mOsml if concerned about renal failure. Serum sodium and serum osmoles are good tests to follow, as a 10 mOsml gradient (296) is needed to get fluid out of brain. (4) Arterial hypotension, sepsis, nephrotoxic drugs, or preexisting renal disease place patients at risk for renal failure with hyper-osmolar therapy. e. Hypertonic saline (HTS) (1) While there is increasing use of hypertonic saline as an alternative hyperosmotic agent, there is insufficient evidence available from comparative studies to support a formal recommendation. (2) The BTF Committee thus chose to re-state the 3rd Edition recommendations. The rationale for doing so is to maintain sufficient recognition of the potential need for hyperosmolar therapy to reduce ICP, while acknowledging that more research is needed to inform more specific recommendations. (3) Effect on ICP is due to the osmotic gradient that moves water across an intact blood-brain barrier reducing the cerebral water content of mainly non-traumatized brain tissue. HS also dehydrates endothelial cells producing vessel dilation and shrinks RBCs that increases their deformability (ability to perfuse through capillaries) leading to plasma volume expansion and improved blood flow. It also reduces WBC adhesion in traumatized brain. (4) Hypertonic Saline (HTS) is recommended for both peds (dose range 6.5 - 10 ml/kg); and adults. (5) Side effects: Risk of central pontine myelinolysis when given to patients with preexisting chronic hyponatremia. Risk of inducing or aggravating pulmonary edema in patients with underlying cardiac or pulmonary problems

Cerebral blood flow

is a function of cerebral perfusion pressure and the brain's ability to autoregulate cerebral blood vessels. Brain injury decreases cerebral blood flow (CBF) while increasing demand for blood and oxygen. CBF following injury may be disrupted by compression of cerebral blood vessels from mass lesions, reduced cerebral metabolism, or to posttraumatic vasospasm as has been documented in as many as 40% of these patients. It can also be reduced due to increased ICP or low systemic BP (hypotension).

Traumatic brain injury (TBI)

"a traumatic insult to the brain capable of producing physical, intellectual, emotional, social, and vocational changes." It is classified as direct (primary) or indirect (secondary) injury to the tissue of the cerebrum, cerebellum, or brainstem. Brain injury affects who we are, the way we think, act, feel and move. It can change everything about us in a matter of seconds

Herniation syndromes; life threatening

1. As mass lesions occupy more volume, intracranial compliance (change in cerebral volume/intracranial pressure) decreases, and elastance (change in cerebral pressure/cerebral volume) increases. 2. A critical threshold is reached when space-occupying lesions can no longer expand without neuronal injury, herniation, and brain death. 3. The brain tissue will treat itself if the pressure is not relieved. Folds of dura compartmentalize the brain. Herniation occurs when increased volume, pressure and/or decreased compliance causes a part of the brain to shift from one compartment into another, causing compression of other structures. The pressure and traction placed on the underlying tissue causes malfunction of that particular cerebral tissue. Death often results.

Alteration in vascular volume, cardiac output, cardiac rhythm, or cerebral perfusion

1. Assess general rate (fast, normal, slow); presence and quality of peripheral pulses, and skin condition. Hypotension must be avoided or corrected immediately to maintain CPP > 60 mmHg. 2. Cold, moist skin suggests hypovolemic shock. Patients cannot lose enough blood due to intracranial bleeding to cause hypotension and shock except in infants. If pulses are weak, thready, tachycardic, or absent at the radials and present at the carotids, attempt to determine the reason. Look for large scalp hematomas or hemorrhage, tension pneumothorax, hemothorax, pericardial tamponade, hemoperitoneum, pelvic fracture, loss into an extremity or retroperitoneum. 3. While undressing the patient, quickly look for obvious wounds or deformities. 4. If patient has AMS and limited BP / perfusion, maintain supine position in ED to enhance CPP as autoregulation may be lost. Do not elevate the head of the bed. 5. Do not let bleeding continue; gain hemostasis as soon as possible. Apply sterile dressing with hand pressure & pressure dressings to open wounds. Topical hemostatic agents may be particularly effective for scalp injuries. Do not apply pressure over a possible unstable or open skull fracture. Apply cold pack. Stabilize impaled objects - do not remove. 6. Cardiac monitor: 90% have dysrhythmias which are brainstem mediated due to release of catecholamines. Obtain 12-L ECG if dysrhythmia present: PACs, SB, SVT, PVCs, VT, Torsades, & VF. SAH: Pathological Q waves, ST elevation or depression; prolonged QTc, wide, large & deeply inverted (neurogenic) T waves; prominent U waves > 1 mm common causing incorrect suspicion of myocardial ischemia. 7. Maintain euvolemia a. Administer IVF to avoid hypotension and/or limit hypotension to the shortest duration possible. Fluid therapy is used to augment cardiac preload, support cardiac output, BP and peripheral O2 delivery in an effort to maintain adequate CPP limiting secondary brain injury. b. The most commonly used IVF is an isotonic crystalloid solution. Administer in quantities necessary to support BP in the target range. Inadequate volume resuscitation can precipitate sudden hypotension and should be avoided. Fluid resuscitation should be done in such a way that does not cause secondary blood loss or hemodilution. c. Avoid dextrose containing solutions unless hypoglycemia is confirmed due to their potential risk of worsening CNS lactic acidosis and cerebral edema. d. Administer volume expanders and blood products as prescribed. e. If a hypotensive trauma patient does not respond to fluids, the brain becomes a secondary consideration.

Ineffective breathing pattern; potential or actual

1. Assess general respiratory rate (very fast or slow), depth, pattern and effort. 2. Assist ventilations as needed with a BVM at 10 breaths/minute for adults; 15-20 BPM for children; and 30 BPM for infants. If long-term ventilatory assist is necessary, prepare the patient for intubation and place on mechanical ventilator. D. Inadequate gas exchange; potential or actual 1. Clinically assess patient for signs of hypoxemia/hypoxia and hyper/hypocapnia. 2. Hypoxia (PaO2 <60) generates free radicals and must be avoided if possible or corrected immediately. Most hypoxia resistant cells: pupils; most hypoxia susceptible cells: temporal lobe. 3. SpO2 should be monitored continuously on all patients with severe TBI. Correct hypoxemia by giving O2 to achieve an SpO2 of 94%. 4. Monitor capnography/ETCO2 (normal = 35-45). EtCO2 increases before ICP goes up. Use as a monitor of pulmonary blood flow, correct placement of an ET tube, and adequacy of ventilations and/or chest compressions. 5. Monitor ABG results: pH, PaCO2; HCO3; BE; PaO2; O2 sat. Maintain normocarbia in the absence of clinical signs of herniation (pupillary dilation or asymmetric reactivity, motor posturing, coma).

Anticonvulsant medications

1. BTF Guidelines (2016): Level IIA • Prophylactic use of phenytoin or valproate is not recommended for preventing late PTS. 2. Phenytoin is recommended to decrease the incidence of early PTS (within 7 d of injury), when the overall benefit is thought to outweigh the complications associated with such treatment. However, early PTS have not been associated with worse outcomes. 3. At the present time there is insufficient evidence to recommend levetiracetam compared with phenytoin regarding efficacy in preventing early post-traumatic seizures and toxicity 4. High risk patients: a. GCS < 10 b. Cortical contusion c. Depressed skull fracture d. Subdural, epidural and intracerebral hematomas e. Penetrating head wound f. Seizure within 24 hours of injury 5. Most current adult recommendation: Only 3 factors have been linked to a high incidence of late seizures: intracranial hematoma, depressed skull fracture and seizure within the first week following injury. Seizure activity AND anticonvulsants can inhibit brain recovery, so they should be used very judiciously 6. In the acute period, seizures may precipitate adverse events in the injured brain due to elevations in ICP, BP changes, changes in O2 delivery, and excess neurotransmitter release. 7. The current recommended drug for TBI seizure prophylaxis is Keppra (levetiracetam); 20 mg/kg IV over 5-15 minutes loading dose followed by 1 gram q. 12 hours times 7 days P. Monitor for and treat hyperglycemia: Hyperglycemia is linked to adverse outcomes after admit for acute neuro events. Q. Narcotic antagonists if sus OD: Naloxone 0.4 to 2 mg IVP

Autoregulation

1. Cerebral autoregulation is defined as the maintenance of cerebral blood flow (CBF) over a wide range of CPPs, brought about by homeostatic changes in cerebral vascular resistance. 2. Thus, assuming that CPP provides the stimulus for cerebral autoregulation, no change in flow would be anticipated as long as the CPP remains within the upper and lower limits of autoregulation. 3. The fluctuating tone of cerebral arterioles depending on the brain's changing metabolic needs, normally maintains a constant blood flow to the brain over a wide range of perfusion pressures. Normal cerebral blood flow is 50 mL/100 Gm/min. Autoregulation functions maximally at a MAP of 60 - 180 mmHg. 4. Metabolic or chemical influences: A change in metabolic rate will lead to a change in CBF. a. pO2 (1) Little effect on CBF when in physiologic ranges (2) Hyperbaric levels and/or hyperoxia some patients result in vasoconstriction (3) Hypoxia increases the severity of any head injury. A pO2 < 60 mmHg (SpO2 < 90) causes cerebral vasodilation, increased CBF, and increased cerebral blood volume. This contributes to increased ICP. (4) PaO2 < 30 doubles the CBF b. pCO2 (1) 1 mmHg change = 2%-3% change in CBF between 20-80 mmHg (2) A pCO2 > 45 (hypercarbia) causes cerebral blood vessels to dilate with corresponding increases in CBF and cerebral blood volume. In the presence of an already high ICP, this extra dilation can have devastating effects. (3) Conversely, low levels of CO2 cause pronounced vasoconstriction that can almost stop perfusion through the brain. This effect will decrease after 6 to 10 hours. (4) Impaired CO2 reactivity impairs O2 reactivity. 5. If autoregulation remains intact, a drop in SBP triggers an autoregulatory vasodilation in an attempt to maintain adequate brain perfusion. This results in increased cerebral blood volume, which in turn elevates intracranial pressure.

Cerebral perfusion pressure (CPP)

1. Cerebral perfusion pressure (CPP) is defined as the pressure gradient across the cerebral vascular bed, between blood inflow and outflow. Inflow pressure is taken as mean arterial pressure (MAP), which by convention is calibrated to the level of the right atrium of the heart. In normal physiology the outflow or downstream pressure is the jugular venous pressure (JVP), which is also calibrated to the level of the right atrium. 2. Traumatic brain injury (TBI) is a special pathological state in which pressure surrounding cerebral vessels—intracranial pressure (ICP)—is elevated and higher than the JVP. In this circumstance CPP will be proportional to the gradient between MAP and mean ICP, and changes in CPP can occur with alterations in either MAP or ICP. 3. Factors that influence CPP: CPP can only be calculated when the ICP is known, and this should be factored into the decision about whether to place an ICP monitor. CPP = MAP - ICP a. Normal MAP = 90-100 mmHg b. Normal ICP in adults: 0-10 mmHg c. Normal CPP = > 60 mmHg but should be patient specific (1) Infants > 50 (2) Children > 60 4. As the ICP rises near the MAP, the gradient for CBF decreases and perfusion is restricted. In a hypotensive patient, even a marginally elevated ICP can be harmful. The body usually compensates for increased ICP by elevating the arterial BP to maintain CPP. 5. Ultimately, the adequacy of CPP is more important than increased ICP. A decrease in CPP results in a reduction in cerebral blood flow. Decreased CPP = altered level of consciousness. Need a minimum CPP gradient of 60 mmHg to be conscious. 6. Never lower the BP in a head trauma patient! a. CPP < 60 = Impaired blood flow to brain b. CPP < 50 = Critical reduction in brain tissue oxygen c. CPP < 40 = CBF down 25% d. CPP ≤ 30 = Irreversible brain ischemia e. If ICP ≥ MAP: The patient is brain dead f. Recommended Threshold: Level III: Maintaining SBP ≥ 100mmHg for patients 50 - 64 years old or ≥ 110 mmHg for patients 15-49 or >70 years of age may be considered to decrease mortality and improve outcomes. 7. Enhancing intravascular hydrostatic pressure by increasing the BP and CPP can help to improve cerebral perfusion. In most cases, CPP is amendable to clinical manipulation, and enhancement may help avoid global and regional ischemia. 8. There is no direct relationship between CBF and ICP. Studies have shown that ICP changes very little when BP is increased by as much as 30 mmHg in head-injured patients, and this is true regardless of autoregulation status. Thus moderate increases in BP, as might be needed to maintain an adequate CPP should not be expected to cause an increase in ICP in most patients.

Clinical signs of ↑ ICP

1. Early to progressive signs a. Pressure is exerted downward. Cerebral cortices and/or reticular activating system and cranial nerves are affected producing the following: (1) AMS: progressive restlessness, confusion, disorientation and lethargy or combativeness; changes in speech or loss of judgment (2) Amnesia of events before or after the injury (3) Increased severity of headaches (4) Visual abnormalities: Diplopia, blurred vision, visual field deficits (lose sight in part of field) (5) Conjugate deviation of eyes or gaze palsies (6) Deterioration in motor function: Monoplegia, hemiplegia. First part of body to show an increase in ICP is the wrist that will over pronate or supinate; pronator drift (7) Sensory loss (8) Oval pupils with hippus (pupil rapidly dilates and constricts when stimulated with light so it looks like it is jiggling up and down) b. Pressure on the hypothalamus: Vomiting (often w/o nausea); temp changes c. Nuchal (neck) rigidity 2. Later signs - game over a. Further alteration in mental status; decreased responsiveness and level of consciousness (coma) b. Pressure on brainstem (1) Cushing's triad (brainstem pressure): (a) Systolic hypertension w/ widening pulse pressure (b) Bradycardia (Vagal nerve pressure) (c) Bradypnea or irregular respirations: pressure on respiratory centers (2) Pupillary changes (unilateral to bilateral dilation) and decreased reactivity to light (CN III paralysis) (3) Further deterioration in motor function: flexor-extensor posturing (non-purposeful movement that is a brain stem reflex) (4) Absent or decreased brainstem reflexes: cough, gag, corneal, Doll's eyes and calorics c. Wide fluctuations in core temperature d. Papilledema e. Seizures

Specific traumatic brain injuries (TBI): Skull fractures

1. Epidemiology a. Involve the cranial vault or basilar skull bones and are classified as linear, vertex or basilar and depressed. Other distinctions include open/closed, stellate, and diastatic (fracture occurs over suture lines). b. Frontal and occipital bones are the thickest. The temporal bone is the thinnest; 50% of fractures occur here. Sphenoid bones and petrous processes of the parietal bone are keystones of the skull base that bear lateral forces when the head is hit. They, and the cribriform plate, are often injured in a basilar skull fracture. c. Only 5% who hit their heads sustain a skull fracture, but 20% with skull fractures had a major head injury. Make brain more susceptible to trauma. d. A compound fracture with neuro impairment >4 hours has a 25X greater chance of deterioration. Skull fracture with obtundation = major head injury; 25% will develop a surgically significant lesion. e. Little growth of new skull bone occurs after two years of age and places patients at risk for post-injury infections. All openings must be covered with sterile dressings followed later by vacuum-assisted closure (VAC); in patients with complex cranial wounds with extensive scalp, bone, and dural defects may use pericranial flap, regenerative tissue matrix overlying CNS tissue, and/or prosthetic grafts.

Volume pressure relationship

1. ICP, while important in itself, must also be considered in the context of its inverse relationship with CPP. CBF is dependent on cardiac output (CO) and is independent of systemic arterial resistance (TPR). If arterial pressure is > 160 torr, cerebral blood volume increases. Initially, as volume increases, there is little or no increase in pressure due to compensation, but as compliance is lost, small additions of volume result in large increases in pressure. 2. There is significant post-traumatic vasospasm as well as changes in pressure and metabolic autoregulation. Cerebral vascular resistance is altered (often increased) by trauma. There is increasing evidence that CBF is typically very low following TBI and, in many cases, may be near the ischemic threshold. A low CPP may jeopardize regions of the brain with preexisting ischemia. CBF in the vicinity of contusions and subdural hematomas is reduced even further than global CBF. 3. To compensate for an elevated ICP, one of the following must happen: a. Blood volume to brain must diminish, b. The body must increase CSF resorption, decrease production, displace fluid downward into the spinal spaces, or c. Brain tissue is displaced (herniation).

Evolution of pathology causing ↑ ICP: pressure + time are big killers!

1. Increase in brain volume a. Mass: Brain tumor, abscess, blood clot, AV malformation; CSF, blood and/or tissue b. Edema (1) Cytotoxic - intracellular (2) Vasogenic - extracellular edema (tumors) (3) Hydrostatic - tissues surrounding ventricles 2. Increase in CSF volume (hydrocephalus) a. CSF is produced in the ventricles by the choroid plexus (20 mL/hr) b. It is reabsorbed in the arachnoid space by the arachnoid villi c. Communicating hydrocephalus (1) Overproduction of CSF (2) Under reabsorption (blood in subarachnoid space) d. Non-communicating (obstructive) hydrocephalus: An obstruction prevents CSF circulation to the arachnoid villi to be reabsorbed 3. Increase in blood volume: Cerebral blood flow is a function of a. Influx pressures (systole) b. Efflux pressure (venous pressure) c. Vascular radius d. Blood viscosity 4. ↑ ICP leads to compression of arteries → cell ischemia → edema 5. In the presence of ischemia/compression of medulla; the body will attempt to fix itself (maintain cerebral perfusion) producing a CNS response called the Cushing response (systolic hypertension, widened pulse pressure, and reflex bradycardia) 6. Respiratory insufficiency; hypercapnia 7. Vasodilation/hyperemia; increased CBF 8. Rapid clinical deterioration and death

Intracranial pressure

1. Intracranial pressure (ICP) is the pressure inside the cranial vault and is affected by intracranial contents. The intracranial volume is generally fixed in an adult (1200-1500 mL) and does not vary. 2. Three intracranial components are noncompressible a. 80%: Cerebral tissue: Brain is 75% H2O; constant blood brain barrier from intact cell wall membranes b. 12%: Cerebral blood volume: Result of cerebral blood flow - 750 mL constant. 80% of brain blood is venous. Head position is critical to maintain venous outflow and to prevent venous congestion. c. 8%: Cerebral spinal fluid (CSF): 125-150 mL is constant. 3. As early as 1783, Monro, Kellie, and other investigators advanced the notion that the volume of the brain is constant. Under normal conditions, any increase in volume in one compartment (brain volume, cerebral blood volume, increased CSF production, and or decreased CSF clearance) must be matched by a similar reduction in another compartment or ICP will rise (Monro-Kellie hypothesis or "No room in the inn theory"). 4. Mass lesions such as tumors, hemorrhagic lesions, cerebral edema, or obstruction of venous and or CSF return can increase ICP. 5. The work of Weed and McKibben demonstrated dramatic changes in the volume of the brain resulting from administration of hypertonic or hypotonic intravenous solutions. Since that time, IV administration of hyperosmolar agents has become routine in the management of intracranial hypertension and herniation syndromes. However, the optimal agent, their optimal means of administration (i.e., dose and bolus vs. continuous infusion), and their precise mechanisms of action continue to be investigated. 6. Intracranial pressure values a. Normal (1) Child: 0-5 mmHg (2) Adult: 0-10 mmHg b. Intracranial hypertension: 15-20 mmHg c. Malignant intracranial hypertension (1) ≥ 20 mmHg sustained for 30 minutes (2) ≥ 30 mmHg sustained for more than 15 minutes (3) ≥ 40 mmHg sustained for more than 2 minutes d. Recommended ICP Threshold for Intervention (BTF Guidelines, 2016): Level III B: Treating ICP >22mmHg is recommended because values above this are associated with increased mortality.

Infection prophylaxis

1. Level IIA • Early tracheostomy is recommended to reduce mechanical ventilation days when the overall benefit is thought to outweigh the complications associated with such a procedure. However, there is no evidence that early trach reduces mortality or the rate of nosocomial pneumonia. 2. The use of PI oral care is not recommended to reduce VAPs and may cause an increased risk of ARDS. 3. Level III • Antimicrobial-impregnated catheters may be considered to prevent catheter-related infections during external ventricular drainage. 4. Risks of infection and aspects of care (Nice to know - directed to ICU) a. Brain Trauma Foundation Guidelines (2016) cite a 40% ventilated-assisted pneumonia (VAP) rate and a 27% ICP monitor infection rate in patients with TBI. b. Risk of infection is affected by the duration of external ventricular drainage, use of prophylactic parenteral antibiotics, presence of concurrent other systemic infections, presence of a suppressed immune system, intraventricular or subarachnoid hemorrhage, open skull fracture, including basilar skull fractures with CSF leak leakage around the ventriculostomy catheter, and flushing of the ventriculostomy tubing c. Ventriculostomy and other ICP monitors should be placed under sterile conditions to closed drainage systems, minimizing manipulation and flushing. There is no support for routine catheter exchanges as a means of preventing CSF infections. There is also no support for use of prolonged antibiotics for systemic prophylaxis in intubated TBI patients, given the risk of selecting for resistant organisms. Early tracheostomy or extubation in severe TBI patients have not been shown to alter the rates of pneumonia, but may reduce the duration of mechanical ventilation. d. If spinal fluid leak, prophylactic administration of Pneumovax may be warranted

Intracranial pressure (ICP) monitoring

1. Level IIB • Management of severe TBI patients using information from ICP monitoring is recommended to reduce in hospital and 2-week post-injury mortality. 2. Recommendations from the prior (Third) Edition not supported by evidence meeting current standards. 3. ICP should be monitored in all salvageable patients with a TBI (GCS 3-8 after resuscitation) and an abnormal CT scan. An abnormal CT scan of the head is one that reveals hematomas, contusions, swelling, herniation, or compressed basal cisterns. 4. ICP monitoring is indicated in patients with severe TBI with a normal CT scan if ≥2 of the following features are noted at admission: age .40 years, unilateral or bilateral motor posturing, or SBP <90 mm Hg. 5. ICP data use a. ICP data can be used to predict outcome and worsening intracranial pathology, calculate and manage CPP, allow therapeutic CSF drainage with ventricular ICP monitoring and restrict potentially deleterious ICP reduction therapies. b. Can be the first indicator of worsening status, evolving mass lesions requiring surgery and hydrocephalus. c. Treatment of ICP without ICP monitoring carries risk (1) Prolonged hyperventilation significantly reduces CBF. (2) Mannitol has a variable ICP response in extent and duration of ICP reduction. 6. Limitations of ICP monitoring: Inability to assess cellular oxygenation

Deep vein thrombosis (DVT) prophylaxis

1. Level III a. LMWH or low-dose unfractioned heparin may be used in combination with mechanical prophylaxis. However, there is an increased risk for expansion of intracranial hemorrhage. b. In addition to compression stockings, pharmacologic prophylaxis may be considered if the brain injury is stable and the benefit is considered to outweigh the risk of increased intracranial hemorrhage. c. There is insufficient evidence to support recommendations regarding the preferred agent, dose, or timing of pharmacologic prophylaxis for deep vein thrombosis. 2. In the absence of prophylaxis, patients with severe TBI are at high risk for developing DVT with embolic events. 3. Methods used for detection: clinical evidence; or Duplex scanning, venography, radiolabeled fibrinogen scans in asymptomatic patients. 4. Treatment in neurosurgical patients is complicated by the uncertainty of the safety of anticoagulant therapy. Prevention is critical. 5. Mechanical prophylaxis agents: Graduated compression stockings and /or intermittent pneumatic compression stockings do not increase BP, ICP or CVP. Lower extremity injuries may limit their use or application may limit ambulation and physical therapy. Foot pumps or pneumatic stockings may be placed on uninjured arms in lieu of legs to achieve a therapeutic fibrinolytic effect. 6. Pharmacological agents a. Low-dose heparin b. Low-molecular weight heparin c. Risks associated with both include intracranial and systemic bleeding that may lead to morbidity and death.

Ventilation therapies Level IIB

1. Prolonged prophylactic hyperventilation with PaCO2 of <25 mm Hg is not recommended. 2. Recommendations from the prior (3rd) Edition not supported by evidence meeting current standards but restated to "maintain sufficient recognition of the potential need for hyperventilation as a temporizing measure." 3. Hyperventilation is recommended as a temporizing measure for the reduction of elevated ICP. 4. Hyperventilation should be avoided during the first 24 h after injury when CBF often is critically reduced. 5. If hyperventilation is used, SjO2 (jugular venous O2 saturation) or BtpO2 (brain tissue O2 partial pressure) measurements recommended to monitor O2 delivery. 6. If hyperventilation is used, ventilate delivering 12-15 mL VT/kg augmented by 15 L O2 to maintain pCO2 between 30-35 torr for as short a time as possible. Allow time for exhalation. If patient fails this regimen, anticipate a poor outcome. Excessive vasoconstriction can worsen cerebral ischemia. 7. Aggressive hyperventilation may cause loss of autoregulation. Slight cerebral hyperemia is probably preferable. Hyperventilation rates a. Adolescents and adults: 16-20 breaths/min b. Elementary school children 30 breaths/min c. Infants and toddlers 35-40 breaths/min 8. Whenever possible, don't hyperventilate or give PEEP to patients in shock as it will increase intrathoracic pressure and decrease venous return and cardiac output. F. Assess for tension pneumothorax, open pneumothorax, or flail chest; resuscitate per Chest Trauma outline if present.

Penetrating (open) trauma

A penetrating injury produces an opening through the skull into cranial contents exposing them to the environment, creating a risk for infection and other injuries. Often caused by missiles such as rifles, hand guns, or shotguns and less commonly by other penetrating implements like knives, ice picks, axes, etc. While not as common as blunt trauma, they are very disruptive due to energy forces that can project hair, skin, bone and debris into the brain and contaminate the region. If the projectile is traveling at a low rate of speed through the skull, it can ricochet within the skull and widen the area of damage. High velocity projectiles can produce significant trauma from shock waves. Sharp projectiles may be superficial because of the protection afforded by the skull, but may pierce through bone and meninges into the brain. A "through and through" injury occurs if an object goes through the skull, brain, and exits the skull. These will produce the effects of penetration injuries, plus additional shearing, stretching, and rupture of brain tissue

Guidelines for assessing and managing patients with TBI

A. The American Association of Neurological Surgeons (AANS) and the Brain Trauma Foundation (BTF) has published patient care guidelines for the management of neurologic injury with widespread distribution since 1996. The CDC conducted a study to assess the effectiveness of adopting the Brain Trauma Foundation (BTF) in-hospital guidelines for the treatment of adults with severe traumatic brain injury (TBI). This research indicated that widespread adoption of these guidelines could result in decreased mortality and significant cost savings, but acutely and long-term B. These have been updated with the most current version published in 2016 available online at www.braintrauma.org The 4th Edition of the guidelines is transitional. They do not intend to produce a 5th Edition. Rather, they are moving to a model of continuous monitoring of the literature, rapid updates to the evidence review, and revisions to the Recommendations as the evidence warrants. C. Prehospital Management of Severe Traumatic Brain Injury D. Field Management of Combat Related Head Trauma E. Acute Medical Management of Severe Traumatic Brain Injury in Infants, Children & Adolescents F. Surgical Management of Traumatic Brain Injury G. Early Indicators of Prognosis in Severe Traumatic Brain Injury H. Classifications of evidence: The levels were primarily based on the quality of the body of evidence as follows 1. Level I recommendations were based on a high-quality body of evidence. 2. Level II A recommendations were based on a moderate-quality body of evidence. 3. Level II B and III recommendations were based on a low-quality body of evidence. I. Significance: These evidence-based guidelines guide care for all brain injuries, regardless of exact mechanism, bony or structural injury. They provide guidance for the CORE ASSESSMENTS and INTERVENTIONS in the brain injured patient. They will be reviewed first, as the core elements of care, followed by specific injury types.

Initial assessment and resuscitative interventions

A. The first two hours post-injury are characterized by ischemia and a 3% decrease in CBF that must be corrected. The first priority is rapid physiologic resuscitation and prevention of secondary injury. No specific treatment should be directed at intracranial hypertension in the absence of transtentorial herniation or progressive neurological deterioration not attributable to extracranial explanations. B. Assess, establish and maintain a patent airway 1. Assume spine injury in all head trauma patients w/ AMS; apply/maintain spine motion restriction while establishing/confirming airway patency. a. Check sizing and effectiveness of spine motion restriction devices applied in the field; maintain until spine is cleared clinically or radiographically. b. Vomiting precautions If the patient is not paralyzed, anticipate vomiting from the head injury or increased ICP with pressure on the medulla, although uncommon in adults. May or may not have associated nausea. (1) Projectile vomiting is due to direct pressure on the medulla or Vagus nerve roots (CN X), which aborts normal sensory pathways and results in violent contractions of abdominal and thoracic muscles. Seen more often in children. (2) Vomiting is especially dangerous in patients with head trauma who have an altered gag reflex as they cannot protect their airways and frequently aspirate. Gastric contents are extremely acidic and will rapidly damage pulmonary tissues leading to a high patient mortality. c. Have large bore suction equipment available at all times.

Secondary (indirect) injury:

All brain damage does not occur at the moment of initial trauma. Secondary injury occurs as a direct result of the primary injury and evolves over minutes, hours and days. Patient outcomes improve when delayed insults are prevented or respond to treatment. Secondary injury is due to a variety of metabolic and physiologic processes initiated by unchecked cerebral edema and regional ischemia. When oxygen or glucose delivery to tissue is limited to the point that tissue needs are not met, metabolism fails and cells die 1. Ischemia: Cerebral ischemia may be the single most important secondary event affecting outcome following severe TBI. Cerebral blood flow during the first day after injury is less than half that of normal individuals even though levels may subsequently increase to normal or supranormal levels. The initial hypoperfusion may cause irreversible damage. 2. Systemic causes a. Hypoxia: Hypoxia, (apnea/cyanosis or PaO2 <60; SpO2 <90%) and hypotension are among the five most powerful predictors of poor outcome, independent of variables such as age, admission GCS, intracranial diagnosis, and pupillary status. There is intense cerebral vasodilation in the presence of hypoxia that adds to an increase in ICP. The temporal lobe is particularly sensitive to oxygen deprivation that results in memory losses due to decreased protein synthesis. The combo of hypoxia and hypotension increases mortality to 75%. b. Hypotension: Almost 1/3 of head injured patients present in shock with hypotension. In the early stages, shock may be more important than hypoxia to treat due to its threat to cerebral perfusion. The injured brain is more susceptible to shock. In a 1993 study, it was found that even one prehospital BP < 90, doubles the patient's mortality! This was reaffirmed subsequently in studies that found a MAP ≤65 mmHg during the first four hours after trauma was associated with a four-fold increase in the odds of non-survival and may have increase the rate of secondary brain injury. c. Electrolyte imbalances (Na, K, Ca) d. Anemia e. Hyperthermia f. Hypercarbia (respiratory acidosis): also causes intense cerebral vasodilation adding to intracranial pressure. g. Hypoglycemia 3. Intracranial causes: Intracranial hypertension, failure of autoregulation, delayed intracerebral hematoma evacuation, edema, structural damage, hyperemia, carotid dissection, seizures, vasospasm, and infection. Large mass lesions result in shifts and displacement of IC contents that result in vascular occlusion, edema, and increased ICP. 4. Tissue surrounding the initial injury is most vulnerable - ischemic penumbra. Secondary injury is often preventable. The "secondary" brain damage opens a window of opportunity where injury and loss of nerve function may be minimized by proper medical treatment.

Mechanisms of injury

Brain injury is usually due to a combination of forces 1. Acceleration: Stationary head is hit by a moving object as in car vs. pedestrian, abuse, or sports injury. 2. Deceleration injury: Moving head hits a stationary object as in falls, abuse, sports injuries and MVCs. Sudden deceleration may produce bony deformity or cause the brain to slide back and forth by 1⁄2 inch at 38 mph collision. The brain can move in a straight linear acceleration with no loss of consciousness but can be injured as it moves across the rough base of the skull. The initial impact and pressure wave may tear tissue and result in injury on the side of the impact (coup) and the side opposite the point of impact (contrecoup). When these forces are applied, shearing, tensile and compressive stresses may lead to fractures, hemorrhage, hematomas, and contusions. 3. Acceleration/deceleration injury: Moving head hits a moving object 4. Distraction injuries: Ex. hanging. If the head is suspended in a drop 18" taller than the person; it causes a fatal blow to the CNS. 5. Penetrating trauma 6. Blast injury: Ex: explosion. Theorize that kinetic energy transfers to the brain's vascular structures, even in the absence of direct impact

Interpreting GCS results

GCS severity distinctions can be debated depending on full physical exam but management of brain injury is often still determined by GCS. (1) GCS 13-15 - Mild head injury: Should be awake with no significant focal deficits. Increasingly, a GCS of 15 is considered a mild head injury. Risk levels for those with a GCS of 15 can be stratified as follows: (a) Low risk: No symptoms or previous symptoms of dizziness, headache or vomiting. (b) Intermediate risk: Loss of consciousness or posttraumatic amnesia. (c) High risk: Severe headache, persistent nausea or more than one episode of vomiting. (2) GCS 9-12 - Moderate head injury: Patient presents with altered sensorium and/or focal deficits but is still able to follow simple commands. Scores within this range represent 10% of all head injuries. Up to 20% deteriorate to coma (ACS, 2018). Should be admitted for observation, even if the CT is normal. Follow-up CT recommended if initial CT abnormal or deteriorate neurologically. (3) GCS 3-8 - Severe head injury: Coma is defined as a GCS ≤ 8. Patients within this range do not follow simple commands after resuscitation and stabilization. They are at risk for secondary brain injury from hypoxia, hypotension, and anemia. Emergent CT scanning is indicated. 2. Pupils for size, shape, equality, reactivity to light 3. Assess for reversible causes of AMS: Hypoglycemia and drug toxicity have been reported as the cause of traumatic events. As with brain injury, hypoglycemia and drug intoxication may present with AMS with or without focal neurologic deficits. It is recommended that patients with AMS of undetermined etiology have a rapid glucose determination. Blood glucose level may be elevated in the presence of intracranial hemorrhage and secondary ischemia. Evidence exists that patients with ischemic brain injury with hyperglycemia (>200) have worse outcomes than those with normal serum glucose levels. An injured brain is hypermetabolic and glucose intolerant. If glucose levels are available, do not give dextrose unless they are hypoglycemic. Use clinical assessments when making treatment decisions. Consider the presence of drug toxidromes that may be reversible.

Specific traumatic brain injuries (TBI): Extracranial scalp injuries

Scalp has five layers and it is well vascularized. The blood vessels do not constrict as well as in other areas of the body, so are at risk for bleeding profusely when injured. Common injuries: 1. Lacerations: Blunt trauma can tear skin and underlying connective tissue causing it to separate 2. Incisions: Smooth wound margins as if cut with a knife. 3. Hematomas: Closed injury causing blood to accumulate within scalp layers. May bleed enough over a depressed skull fracture to fill depression and conceal injury. 4. Abrasions, contusion, burns 5. Avulsions: Scalp tissue is only loosely attached to the skull. Shearing forces may tear a flap of tissue, exposing a portion of the skull. Risk for serious contamination and bleeding. 6. A scalp hematoma or laceration may suggest deeper injury beneath - have a high index of suspicion for skull fracture. Danger: blood loss sufficient to cause shock. Control bleeding as soon as possible.

Blunt (closed) trauma

The person receives an impact to the head from an outside force, but the skull and dura remain intact and brain tissue is not exposed to the environment. More common than penetrating. The structures of the head and face generally protect well against most blunt trauma. However, when the magnitude of forces exceeds the tensile strength of the structures, severe injury can occur. For example, the sinus cavities of the face are frequently injured with blunt facial trauma. The air-filled spaces collapse upon impact and help to dissipate energy forces. A person may have a closed head injury with mild to severe traumatic brain injury.

Steroids: BTF Recommendations Level I

The use of steroids is not recommended for improving outcome or reducing ICP. In patients with severe TBI, high dose methylprednisolone was associated with increased mortality and is contraindicated

How the brain is injured

There are two distinct phases of injury that produce neurological dysfunction to the tissues of the cerebrum, cerebellum, or brainstem: Primary (direct) injury and Secondary (indirect) injury.

Focused neuro exam: Cranial nerve exam

a. I: Olfactory - ability to smell b. II: Optic - Visual acuity; sensory limb of pupillary light reflex c. III: Oculomotor - Moves eyes up, down, in to nose; lifts eyelid; regulates pupil size, shape, reactivity d. IV: Trochlear - innervates superior oblique muscle e. V: Trigeminal - Sensation to face; motor to muscle of mastication f. VI: Abducens - Pulls eye to ear (III, IV, VI: EOMs and gaze palsies) g. VII: Facial - Motor to muscles of facial expression; sensory to tongue h. VIII: Acoustic vestibular (Vestibulocochlear) - Auditory reception/ balance i. IX & X: Glossopharyngeal & Vagus: Examine together; lifts palate, provides gag reflex j. XI: (Spinal) Accessory - Turns head, shrugs shoulders k. XII: Hypoglossal - Supplies most extrinsic tongue muscles If speaking normally, don't test in isolation.

Specific traumatic brain injuries (TBI): Diffuse injuries: Concussion

a. Incidence: 2-4 million/yr - most common head injury b. Definition: Traumatic, microscopic damage to cells deep in the white matter of the brain causes reversible physiologic disturbance of neurological function that occurs at the instant of trauma. c. Pathogenesis (1) Linear motion of the brain "stuns" the affected cells and puts them on "idle". While in this state, the cells are not dead, but they do not function as they should. (2) Traumatic forces cause transient ischemia, neural depolarization following sudden acetylcholine release, or microscopic axonal disruption of fibers in upper brain stem (ascending reticular activating system) and temporal lobe producing the classic S&S. (3) Patients don't have to hit their head or lose consciousness (4) They may report seeing stars or flashes of light. (Note: seeing stars is K+ related. Linear motion causes K+ to run out of cells and short circuits the occipital lobe.) (5) Most neurons stain blue. Neurons in temporal lobe following a concussion stain red from damage. d. Clinical presentations: Suspect if one or more of these are present: (1) Physical symptoms: Headache, dizziness or vomiting (2) Physical signs: Ataxia, unsteadiness (3) Impaired brain function: Anterior aspect of temporal lobe frequently impacted area - amnesia is a hallmark sign; confusion cognitive impairment; abnormal behavior e. Post concussive syndrome (PCS) (1) Measurable deficits in cognition and memory generally resolve in 1 month, but in ~30% symptoms persist for ≥ 3 months. Concussions are cumulative. Memory cells in temporal lobe are most subject to damage. Repetitive concussions will cause recent memory loss. (2) The Am Psychological Assoc defines PCS as quantifiable deficits in memory or attention and the onset or worsening of any three of the following: tiring easily, disordered sleep, headaches, vertigo/dizziness, irritability, anxiety/ depression/affective lability, changes in personality, or apathy. Thirty to 70% of patients who have experienced a TBI complain of sleep problems, and the degree of the injury does not predict the severity of symptoms. Studies show 20% still had significant sleep disturbances 3 years after a concussion. The most common sleep-wake disorders affecting those with TBI are insomnia, hypersomnia, pleiosomnia (excessive daytime sleepiness or the need for an unusual amount of sleep in a 24- hour period), sleep-related breathing disorders, circadian rhythm disorders, parasomnias, and movement disorders. (3) May also have problems with sunlight, coffee, nervousness, irritability, negative energy syndrome, difficulty with abstract thinking and judgment, loss of inhibition and libido, and avoidance of crowds. Neuropsychological deficits may be detectable after resolution of neuro symptoms. Recovery may take 6-12 mos. (4) Preinjury factors (age, education, emotional adjustment) and post-injury factors (pain, family support, and stress) interact with cognitive functioning and significantly affect recovery from TBI. (5) Return to work /driving / sports and play are individualized based on clinical status and potentially, formal neuropsychological testing. (6) Exercise: A 2016 Brain Trauma Foundation study reported that rest was over-rated and exercise actually improved post concussive symptoms in all patients, specifically, visual memory and processing speed. Physical rest >1-2 days worsened symptoms, including visual memory and reaction time. f. Diagnostics: Non-contrast head CT using standard criteria. Caution with pediatric patients to avoid unnecessary radiation exposure. MRI, PET, SCEP scans are not recommended.

Cerebral metabolism

The brain is only three pounds of tissue (2% of body weight), but is the most metabolically active and perfusion-sensitive organ. It metabolizes 25% of the body's glucose, burning 60 mg/min. It consumes 20% of the cardiac output and 20% of the total body oxygen (49 mL/min). It has no storage mechanism for O2 or glucose so brain tissue is dependent on an on-going source of both fuels via a constant source of cerebral blood flow (CBF) via the internal carotid and vertebral arteries and will demonstrate altered mental status (AMS) within moments of a reduction in either.

Types of ICP monitoring systems

a. Intraventricular fluid-coupled catheter with an external strain gauge transducer or catheter tip pressure transducer is the most accurate, low cost, and reliable monitor and is the established reference standard. The monitor is set to a prescribed maximum pressure and an alarm sounds when that pressure is reached. It can be recalibrated in situ. ICU nurses are authorized to drain CSF for about 10 seconds to relieve the pressure. An external transducer must be consistently maintained at a fixed reference point relative to the patient's head to avoid measurement error. b. ICP transduction via fiberoptic or micro strain gauge devices placed in ventricular catheters provide similar benefits but at a higher cost. c. Parenchymal: Camino catheter: Closed fiberoptic system placed using micro strain pressure transducers through 18 gauge needles, read tissue pressures and provides results similar to ventricular ICP. Monitoring at 30° is best.. Have the potential for measurement differences and drift due to the inability to recalibrate. NOTE: Catheter is incompatible with MRI. d. Less accurate (1) Subdural devices: Catheter tip pressure transducer of fluid- coupled catheter with an external strain gauge. (2) Subarachnoid: Fluid-coupled device with an external strain gauge (3) Epidural devices

Focused neuro exam: Motor exam

a. Assess spontaneous, purposeful movement, muscle tone & strength to gravity and against resistance. Have patient extend arms in front of them with palms up and eyes closed. Watch for pronator drift to indicate unilateral weakness. This is a more sensitive sign for hemiparesis than grip strength which assesses strength of the forearm or hand muscles. b. Conscious patient: Assess ability to shrug shoulders, flex and extend elbows and wrists, hold fingers spread open against resistance and to open fingers against resistance; flex and extend knees, plantar and dorsiflex feet; wiggle toes. c. If unconscious: Observe spontaneous or purposeful movements and then begin applying stimuli from least noxious to pain. Assess for weakness by holding up both upper extremities and releasing them simultaneously. A weak extremity will fall more quickly. To check lower extremities, bend the knees and place feet together flat on the cart. Hold the knees together and release at the same time. A weak or paralyzed limb will fall immediately. d. Grading function (1) 5 = normal muscle strength (2) 4 = normal range of motion against some resistance (3) 3 = normal range of motion against gravity only (4) 2 = weak contraction; unable to overcome gravity (5) 1 = slight muscle contraction; no joint movement (6) 0 = complete paralysis e. Movement/strength abnormalities (1) Paresis (weakness) (2) Paralysis or -plegia (inability to move at all) (3) Posturing: Abnormal flexion or extension. (4) Clonus: Spasm in which contraction and relaxation alternate in rapid succession f. Muscle tone abnormalities (1) Decreased: flaccid/atonic (2) Increased: spasticity, rigidity

Levels of intracranial pressure with corresponding S&S

a. Cerebral cortex and upper brain stem involved (1) BP rising and pulse rate begins to slow (2) Pupils still midsize and reactive (3) Cheyne-Stokes ventilations (4) Initially tries to localize and remove painful stimuli, eventually withdraws then abnormal flexion occurs (5) All S&S should be reversible at this stage b. Middle brain stem (pons) involved (1) Wide pulse pressure and bradycardia (2) Pupils pinpoint to mid-size, sluggish or non-reactive (CN III) (3) Central neurogenic hyperventilation (4) Abnormal extensor posturing (5) Few patients return to normal function from this level c. Lower portion of brain stem (medulla) involved (1) Pupils both dilated and non-reactive (2) Respirations ataxic or absent (3) Flaccid, does not react to pain (4) Irregular pulse rate (5) QRS, ST, and T wave changes (6) Decreased BP (7) Not considered survivable

Specific traumatic brain injuries (TBI): Focal injuries: Intracerebral hematomas

a. Definition: A deep contusion or tear in blood vessels that result in hemorrhage in the brain parenchyma b. Pathogenesis: Single or multiple hemorrhages may occur from blows to the skull, rotational acceleration, missile-type injuries, and coup-contrecoup lesions. Sixty percent are associated with skull fractures, especially depressed fractures, and 60% appear under contusions. The majority (85%) are located in the frontal and anterior temporal lobes at the anterior base of the brain. They may take some time to develop. Sudden deceleration or head rotation causes these regions to impact the rough surface of the basilar skull. c. Morbidity/mortality: Depends on size. Majority of deterioration occurs within first 48-72 hours. Delayed hemorrhage has been reported 1 to 7 days after injury.. d. Clinical presentation: Similar to contusions; depends on location. Will often share S&S of a stroke. (1) Headache (2) Deteriorating consciousness to deep coma (3) Contralateral hemiplegia (4) Ipsilateral dilated pupil (5) Speech deficits e. Diagnostic radiography (1) CT can determine the volume of blood. If≥ 35-40 cc or if they are located in the posterior temporal lobe → OR (2) Cerebral arteriography f. Surgical indications: Posterior fossa lesions causing significant mass effect. Indications are not clearly defined for supratentorial lesions.

Specific traumatic brain injuries (TBI): Focal injuries: Brain stem hemorrhage

a. Definition: Bleeding into the midbrain, pons, or medulla b. Pathogenesis (1) Primary: Direct impact or torsion injuries of the brain stem (2) Secondary: Compression from ↑ ICP, cerebral edema, or laceration of temporal lobes c. Clinical presentation (1) Midbrain: Deep coma, midpoint fixed pupils, ophthalmoplegia, abnormal extension response to pain (2) Pons: Coma, pin-point pupils, ophthalmoplegia, abnormal extension response to pain (3) Medulla: Pupils bilaterally dilated and fixed; death d. Morbidity/mortality: Almost 100% if comatose e. Diagnostic radiography: CT; MRI

Specific traumatic brain injuries (TBI): Focal injuries: Epidural hematomas (EDH)

a. Definition: Blood clot between the inner table of the skull and the dura that doesn't involve the brain. b. Incidence: Seen in 1%-4% of patients with head trauma, but account for 10% of mortalities. They are more common in young people and rare for the elderly or those <2yrs due to the tightly adherent brain / dura relationship in those age groups. c. Etiology: Falls, direct blows to the head, MVC, sports injuries d. Pathogenesis: (1) Classically, epidurals are unilateral and supratentorial, associated 85% of the time with a linear fracture in the temporoparietal area that can tear the main trunk of the middle meningeal artery (which is encased in a groove of the skull bone after the age of 3). They can occur in other sites due to brisk oozing of blood from diploic vessels (those contained in the spongy middle layer of skull bone) injured by skull fractures. They may be venous in origin if fracture crosses a dural sinus (usually under a skull suture line). (2) Because the dura attaches to the skull at suture lines, an epidural hematoma does not cross suture lines. This creates a contained clot that is lens- or football-shaped on CT. The clot size depends on the rate of bleeding, time from injury to presentation, severity of injury, and clot formation. (3) There is no room for extra bleeding inside the skull, so intracranial pressure rises rapidly. (Remember the Monro: Kellie hypothesis?) The clot will compress cerebral structures (shutting them down) and may eventually shift the brain onto midline structures (brainstem and cranial nerves). e. Morbidity/mortality data: 10% adult and 5% children. With prompt neurosurgical intervention a good prognosis is possible. Poor outcomes are related to a delay in surgical intervention. Higher mortality rates are seen if associated with low GCS, intradural lesions, temporal location, large hematoma volumes, rapid clinical progression, pupil abnormalities, increased ICP and advanced age. f. Peds considerations: Clots not common in children but epidurals occur more frequently than subdurals; except in infant - non-accidental injury. Intracranial blood loss may precipitate shock in young children < 3 years. More common bilaterally in children. May tend to watch longer before operative intervention. g. Elder considerations: Extremely rare in the elderly. h. Co-morbidity: Prevention of secondary injury is critical as it results in cell necrosis and poor outcomes, including death.

BTF Guideline for CPP

a. Level II B: • Management of severe TBI patients using guidelines-based recommendations for CPP monitoring is recommended to decrease 2-week mortality. b. Level II B: The recommended target cerebral perfusion pressure (CPP) value for survival and favorable outcomes is between 60 and 70 mm Hg. Whether 60 or 70 mm Hg is the minimum optimal CPP threshold is unclear and may depend upon the patient's autoregulatory status. c. Level III: Avoiding aggressive attempts to maintain CPP above 70 mm Hg with fluids and pressors may be considered because of the risk of adult respiratory failure. d. For Peds (0 - 18 yo), 40 mmHg minimum goal with range of 40 - 50.

Specific traumatic brain injuries (TBI): Focal injuries: Traumatic subarachnoid hemorrhage (SAH)

a. Definition: Blood in the subarachnoid space between pia & arachnoid meningeal layers. b. Pathogenesis: Traumatic SAH is usually diffuse, does not form a definite hematoma and does not create a mass effect. Bleeding occurs from superficial cortical vessels, an AV malformation, or leaking intracranial aneurysm and is often associated with subdural hematoma. Causes swelling around the brain stem (not good). c. Morbidity/mortality data: If ventricular system is filled with blood, outcome is poor. d. Clinical presentation (1) Severe headache - Rapid onset, intense pain (worst headache of their life) becoming occipital, aggravated by any movement, worse lying down. Can mimic migraine and tension headaches. A majority of non-traumatic SAH patients have a premonitory leak and headache days to weeks before major rupture with neuro damage. (2) Optic fundi have pre-retinal hemorrhages; visual disturbances (3) Ipsilateral dilated pupil (4) Meningeal irritation signs: nuchal rigidity requires 12-24 hours to develop and is not a reliable early sign (5) Deterioration of mental status (6) Hemiparesis; R/O c-spine injury (7) Seizures e. Diagnostic radiography: CT; CTA; arteriography. CT changes may not appear for up to 4 hours (longer with a minimal bleed). Appear as high-density fluid collections within the cerebrospinal fluid spaces. Can be focal or diffuse. f. Emergency interventions (1) Vasospasm risk, secondary to brain irritation, peaks at 7-10 days after injury. (2) ICP monitoring may be indicated (3) Surgical evacuation, nimodipine (60 mg q. 4 hours PO or per NG tube) for vasospasm, and standard treatment for cerebral edema g. Complications: Extensive SAH = risk for communicating hydrocephalus. May need temporary ventriculostomy or LP shunt.

Specific traumatic brain injuries (TBI): Focal injuries: Cerebral contusions (Intra-axial or within the brain substance bleeding)

a. Definition: Ill-defined collections of blood along the brain surface or within the brain substance. Typically seen in the gray matter but may extend into the subcortical white matter. Classified as coup (found at the site of impact), contrecoup (in a line directly opposite the point of impact which is the worse of the two impacts), gliding, and petechial. May involve laceration of vessels and brain tissue with tissue necrosis, pulping and infarction. With most contusions, there is any area of brain cells that are destroyed and will not regenerate. But the surrounding tissue (penumbra) that is being affected by the swelling can be saved with appropriate management. b. Pathogenesis: Temporal and frontal lobes are the primary sites of coup lesions. The frontal lobes bang on the frontal bone. Temporal lobes hit the middle cranial fossa (sphenoid wings) and start to bruise and swell. After the initial impact, the brain "sloshes" backward, again impacting the internal skull structures. If hit on the forehead, the contrecoup lesion is often in the occipital lobes producing visual disturbances (seeing stars). Often multiple and occur in combination with other lesions. The blood-brain barrier in the area of the contusion may lose its integrity, which can lead to the development of an intracerebral hematoma. Contusions and hematomas that are initially small may increase in size causing rapid worsening of a previously stable patient's condition. c. Morbidity: Worsens with ICP ≥ 20-25 mmHg. Significant due to associated hemorrhage, edema, and brain swelling. d. Clinical presentation: Depends on involved structures but often presents with localizing personality, behavior, motor, speech. memory, or visual deficits. Temporal lobe contusions especially severe due to close proximity to the midbrain. Contusions typically worsen over the first 24-48 hours post- injury secondary to inflammatory effect causing further patient deterioration. e. Diagnostic radiography: CT (anticipate serial scans due to above) f. Emergency interventions: Monitor patient with small lesion (<5-7 mm of brain shift on CT) for ↑ ICP. Extensive contusion with hemorrhage and mass effect require surgery.

Specific traumatic brain injuries (TBI): Focal injuries: Impalement/penetrating injuries

a. Definition: Open head injuries due to a projectile b. Pathogenesis (1) Bullets and other projectiles destroy a path of brain tissue along their trajectory (primary injury). Trauma to cerebral vessels may cause large hematomas. Bone fragments, imploded debris, and the breach in cranial integrity may result in the development of infections and cerebral abscesses. Large fragments that lodge in the ventricles or on the brain surface may migrate with catastrophic results. (2) High velocity injuries: Large shock waves and zones of negative pressure result in significant cavitation. Swelling of adjacent tissue may result in fatal ↑ICP. c. Morbidity/mortality: High (50%) due to structural damage, massive brain swelling and uncontrolled ↑ ICP d. Clinical presentation: Depends on sites of injury e. Impaled objects: Patient presentation depends on the structures involved. Some patients are awake and aware with rather large objects impaled into their skull. Do not move or remove. f. Diagnostic radiography: CT, CTA or Angiography recommended. Skull films no longer essential. g. Emergency interventions: (1) "Objects that penetrate the intracranial compartment or infratemporal fossa must be left in place until possible vascular injury has been evaluated and definitive neurosurgical management is established. Disturbing or removing penetrating objects prematurely may lead to fatal vascular injury or intracranial hemorrhage." (2) Patients with small caliber entrance wounds, no major intracranial injury and 'healthy' scalp tissue can be managed appropriately with local wound care and closure. (3) Prophylactic antibiotics are recommended; anticonvulsants may be appropriate.

Specific traumatic brain injuries (TBI): Diffuse injuries: Diffuse axonal injuries (DAI)

a. Definition: Shearing, tearing, or stretching force on nerve fibers at the junction of the gray and white matter causing widespread disruption of neurologic function. Characterized by white matter degeneration as axon is disrupted close to cell body resulting in progressive abnormalities. Lesions are typically found in four areas: corpus callosum, corticomedullary junction, upper brainstem, or basal ganglia. b. Pathogenesis: During rotation, mechanical forces act on the long axonal fibers and cause them to experience structural failure as they are physically separated into distal and proximal segments along with injury to their small accompanying blood vessels. The distal segment degenerates resulting in profound deficits. These injuries can produce diffuse cerebral edema but may also appear as multiple distinct hemorrhagic foci varying from punctate to a few centimeters in diameter. See reactive astrocytes 4-6 hours after the injury. Dead neurons stain red and can never recover. c. Morbidity/mortality: Results in global neurologic dysfunction. Coma is due to brain stem dysfunction. Accounts for 1⁄2 of all admissions and 2/3 of all TBI deaths. May have residual neurologic deficits. Recovery is dependent on the amount and location of neuronal damage. Currently no effective treatment for severed axons. d. Clinical presentation (1) Mild DAI (Grade 1): Widespread axonal damage in the parasagittal white matter of the cerebral hemispheres and is found in those who have a brief loss of consciousness and do not experience come or those with milder injuries. Mortality 15%. (2) Moderate DAI (Grade 2): Tissue tear hemorrhages and axonal abnormalities in cerebral hemispheres and corpus callosum. Coma ≥24 hrs, may have abnormal posturing. Mortality 20%-24%. (3) Severe DAI (Grade 3): Grade II findings plus abnormalities in the upper brain stem, and an increased degree of axonal hemispheric abnormalities. Point of maximum stress where tissues of different density interface (between gray and white matter). Coma is immediate, lasting longer than 24 hours with positive brain stem signs (posturing) and incomplete recovery. Mortality 60% due to direct hemorrhage in brain stem. (4) Classic picture: For patients with milder injuries, the clinical presentation can be unclear. In more severe injuries, immediate, deep, prolonged coma may last weeks to months, accompanied by ↑ ICP, persistent brain stem reflexive posturing (flexion or extension), hypertension, ↑ temperature (104°-105° F), and hyperhidrosis indicating diencephalon involvement. No mass lesions or S&S on CT; no brain stem compression - so no pupil signs. May be associated with a persistent vegetative state. (5) Interventions: Support ABCs. At present, there is no effective curative treatment for DAI. If an axon is injured but not severed, providing an optimum environment for healing may enable it to recover. If secondary injury occurs the loss will be permanent. e. Diagnostic radiography (1) CT may show characteristic small hemorrhagic lesions in the corpus callosum, superior cerebellar peduncles, or periventricular region. Only 1⁄4 initially present with visible hemorrhage, thus a negative CT does not rule out this diagnosis (2) Visualized better on MRI with contrast using gradient-refocused imaging. This technique is highly sensitive to the presence of iron within hemoglobin.

Signs of herniation

a. GCS decreases by 2 or more points b. BP and ICP shoot up c. HR drops to 30-40 d. Respiratory pattern changes e. Pupillary changes (unilateral to bilateral dilation and non-reactive to light). Pupil first dilates on same side as the clot (ipsilateral dilation) then both may dilate and become nonreactive f. Motor abnormality (contralateral deficits) to extensor motor posturing g. Brainstem dysfunction 6. Outcomes a. If medulla brainstem function is altered, the BP will drop to 40-20. HR will increase to 140-150. The brain will not go back to its normal alignment. This is an indication for temporary hyperventilation. b. Generally poor outcomes in the presence of hypoxia, hypotension, hypercarbia, hyperglycemia (which usually means a clot), or increased Ca levels. Check Mg; Mg runs with Ca.

Focused neuro exam: Cerebellar exam

a. Have patient rapidly turn their hands palm up and palm down (rapid alternating movements), rotate their hands in concentric circles (posting), touch their finger to your finger (light on an object), and run the heel of one foot down the shin of the opposite leg. b. An inability to smoothly start and stop or coordinate motion is called ataxia and may indicate cerebellar dysfunction

Expose

examine and recover to maintain warmth

GCS parts

f. Best eye opening: Assesses both wakefulness, arousal mechanisms in the brainstem, and content of behavioral response. Ask the patient, "What happened to you?" If the patient opens his eyes, then ask the questions in the verbal and motor section of the GCS to determine the total score. (1) If the patient does not open his or her eyes, apply a central pain stimulus. Pinch the earlobe or apply pressure over the supraorbital ridge for at least 15-30 seconds if the patient is slow to respond. If the patient is spontaneously moving all four extremities, apply blunt pressure to the nailbed. Alternative method of appropriate pain stimulus: pinch muscles over top of shoulder. (2) Eye opening to command or speech is a higher level of stimulus recognition and one can assume that the cerebral cortex is processing information. 4 Spontaneous: Eyes are open before stimulus. Indicates an intact arousal mechanism. 3 To sound: Eyes open after spoken or shouted request. 2 To pressure: Eyes open after fingertip stimulus to fingertip, trapezius or supraorbital notch 1 No eye opening: No opening at any time, no interfering factor NT Not testable/confounding variables; Chemically sedated or paralyzed/or eyes swollen shut/orbital trauma; CN injury. g. Best verbal response: Assesses the eloquent cortex (where we speak) by evaluating the content and context of speech. While this evaluates a high level of cognitive function, its absence does not imply a total loss of function. This is a difficult area to score with consistency, especially between the options of 4 and 5. 5 Oriented: Correctly gives name, place and date and situation. 4 Confused: Not oriented but communicates coherently - responses may be inaccurate. 3 Words: Intelligible single words - may repeat random, repetitive words, numbers, or profanity. 2 Sounds: Only moans / groans, or cries 1 None: No audible response, no interfering factor NT Not testable/confounding variables: Intubated, paralyzed, sedated, intoxicated or chemically impaired, maxillofacial trauma, grossly swollen tongue, cricothyrotomy, tracheotomy, mutism, aphasia, hearing loss, language barrier, dementia, psych disorders. h. Best motor response: Assesses both arousal and content of behavioral response. This element is the least affected by trauma. It allows evaluation of interface between sensing a stimulus, interpreting the information and reacting to it. Note best response, even if seen only unilaterally. 6 Obeys commands: Ask a conscious patient to move fingers, toes or an extremity. If limbs are paralyzed, ask patient to blink eyes in response to a command. 5 Localizing (Protective response); Localizes pain stimulus and attempts to remove it or move away from it with purposeful movement. This is best assessed by pinching the trapezius muscle, ear lobe or applying supraorbital pressure and observing if the patient tries to move your hand away or remove the pressure source. The hands should move across midline or above the nipples to confirm purposeful movement. Examples: pt. tries to remove a c-collar or oxygen mask; moves the arm in which a pain stimulus like an IV start or blood draw is being done. This response indicates that the parietal lobe is functioning to interpret and localize the stimulus and that it can communicate with the motor cortex in the frontal lobe for purposeful movement. 4 Normal flexion: Generalized purposeful movements pulling both arms in toward the torso. Pt. perceives pain, but cannot localize the stimulus. This response indicates that pain pathways to the thalamus are intact, but the parietal lobe is not interpreting or localizing the pain source. 3 Abnormal flexion (old decorticate posturing): Adduction of upper extremities with flexion or the wrist or elbows and extension of the legs in non-purposeful, reflexive movement. Indicates lesions in the cerebral hemispheres or internal capsule 2 (Abnormal) Extension (old decerebrate posturing): Adduction, hyperpronation and extension of upper extremities, internal rotation of shoulders, extension and plantar flexion of the legs in nonpurposeful movement. May progress to arching of the back (opisthotonos). Reflects midbrain to upper pontine damage. Posturing is a brain stem reflex. 1 None; no response (movement); no interfering factor NT Not testable/confounding variables: Chemically or traumatically paralyzed, peripheral nerve injury, extremity trauma with immobilization, pain, inability to comprehend commands, dementia, psychiatric disorders, alcohol/ drug intoxication.

Specific traumatic brain injuries (TBI): Skull fractures: Vertex fractures

a. Linear (1) Definition: An inbending of the skull at the point of injury with simultaneous outbending around the region of impact which dissects both tables of bone causing a crack. (2) Etiology: Low-velocity, blunt or compression trauma (3) Incidence: 80% of all skull fractures. Occur most frequently in children and elderly. (4) Pathogenesis: 50% involve temporal and parietal bones (5) Morbidity/mortality (a) Most are essentially benign (b) A diastatic (sprung suture) is the biggest predictor of who will deteriorate from a linear fracture (c) A fracture over the temporal/parietal bones should cause suspicion for an epidural hematoma (6) Clinical presentation: External soft tissue trauma, subgaleal hematoma, and pain (7) Diagnostic radiography: standard skull film or CT (8) Emergency intervention: Usually requires no treatment. If fracture extends into orbit, paranasal sinus or crosses a major vascular channel, admit to observe. Fracture line usually disappears in 6 months (children) and 3-4 years in adults. b. Stellate/comminuted (1) Definition: Skull fractures into multiple fragments that may penetrate the meninges and damage structures beneath (2) Etiology: Moderate-velocity blunt or compression trauma c. Open: See penetrating injuries (1) Definition: Combination of a depressed skull fracture and a scalp laceration. The dura may be torn. (2) Pathogenesis (a) Blunt and/or penetrating trauma (b) AK 47 with Teflon bullet produces 2100 lbs/in2 of force that explodes the inside of the head. (3) Morbidity/mortality: High (4) Clinical presentation: Depends on site (5) Diagnostic radiography: Standard skull films; CT (6) Emergency intervention: OR d. Depressed (1) Definition: Fracture with inward displacement of a bony segment (2) Etiology: High velocity contact over a small surface area results in compression trauma (3) Pathogenesis: Results from direct applied forces. Frequently seen in MVCs and violence. Ex: Hammer creates 1600 lb/in2. (4) Classifications (a) Closed with scalp intact (b) Compound with scalp open but dura intact (c) Complex with scalp and dura lacerated by bone fragments = cortical laceration and hemorrhage. (5) Morbidity/mortality: Causes intracranial damage like contusion and laceration; infections and seizures (6) Clinical presentation: Can usually see or feel depression unless covered by a scalp hematoma. May have focal brain injuries. (7) Diagnostic radiography: standard skull films, CT (8) Emergency intervention: If open, greater than 1 cm depressed, or associated with neurological deficit: surgical debridement, evacuation of clots and bone fragments, and elevation of depressed bone and repair of lacerated dura. Prophylactic anticonvulsants and antibiotics. e. Growing fracture - See peds module

Types of herniation

a. Supratentorial herniation (1) Cingulate herniation: Cingulate gyrus herniated into the falx (2) Uncal herniation: Medial edge of the temporal lobe herniates down through the tentorial notch into the posterior fossa. Ipsilateral pupil dilates and fixes. Contralateral motor weakness or paralysis. (3) Transtentorial (central) herniation: Downward displacement of the cerebral hemispheres, diencephalon, and midbrain through the elongated tentorial notch. Both pupils are fixed and dilated; bilateral motor posturing. b. Infratentorial: Cerebellar (tonsillar) herniation - Cerebellar tonsils herniate into the foramen magnum. Causes almost immediate death.

Advanced airway adjuncts

a. The high incidence of hypoxia and risk of aspiration in patients with TBI have been used to justify an aggressive approach to airway management, including intubation. b. Of concern, there is a threefold increased odds of mortality in those that were intubated in the field after adjustment for injury severity and other confounders. The high incidence of hyperventilation following intubation, desaturation and bradycardia during tube placement, and undetected esophageal intubation may be important contributors to increased mortality in intubated patients. Laryngoscopy without benefit of neuroprotective drugs may be harmful in this subset of patients. Positive intrathoracic pressure caused by over ventilation may decrease venous return and impair cardiac output in hypovolemic patients. High intrathoracic pressures can also cause an increase in ICP.

Head Injury

defined as external influences causing traumatic insult to the head that may result in injury to soft tissue, bony structures and/or brain.

Specific traumatic brain injuries (TBI): Diffuse injuries: Concussion Continued

g. Concussion management tools: (1) The computerized Immediate Post-Concussion Assessment and Cognitive Testing (ImPACT) system and ImPACT Pediatric are both manufactured by the ImPACT Applications company. (a) Created for people 12 to 59 years who have sustained a head trauma. Using a desktop or laptop computer, licensed clinicians use specialized software to assess a patient's cognitive skills, such as working memory, attention span, nonverbal problem solving, and reaction time. The results are matched to a control-group database or, if possible, a patient's own previous scores. (b) The child-specific ImPACT Pediatric works the same way but is for patients aged 5 to 11 years and works only on an iPad. It has a game-like design and takes about 10 to 15 minutes to complete. (2) SCAT 3 (Sports Concussion Assessment Tool; 3rd edition) is used by Sports Medicine and Athletic Trainers in Illinois to assess athletes' readiness to return to play. It addresses symptoms, memory, concentration, balance, upper extremity coordination and delayed recall. h. Emergency interventions (1) Reorient patient to time and place (2) Analgesia for headache (3) Prophylactic anticonvulsants controversial - not generally used (4) Discharge from the ED is warranted if concussion is isolated, there are no other injuries and there is reliable ongoing monitoring available. (5) Patient /family education is important to identify expected behaviors & those warranting emergent return for evaluation.

Specific traumatic brain injuries (TBI): Focal injuries: Acute subdural hematomas Continued

g. Morbidity/mortality: 40-60% overall mortality; rises to 57%-68% if the patient presents in come. Generally fatal if the clot comprises >8-10% of intracranial volume. (1) Poor prognosis due to primary brain damage underlying the clot, vascular injury, hyperemic response leading to increased intracranial volume, ipsilateral hemispheric or total brain swelling, and ↑ ICP. (2) Factors predictive of functional recovery (a) Evacuation within 4 hours of injury (if applicable) (b) Post-op ICP < 20 mmHg (c) Normal post-op evoked potentials (d) GCS > 5 i. Diagnostic radiography: Looks like a crescent-or sickle-shaped collection of blood between the inner table of skull and the brain parenchyma on CT. May co-exist with an epidural hematoma. j. Emergency interventions: Surgical evacuation of clot if >10mm thick with associated 5mm shift, hemorrhage control, or need for resection of the contused, nonviable brain. k. Nonoperative management is a viable option even if the patient presents in coma unless the above parameters are met. Symptomatic patients with sub-acute and chronic lesions are generally managed non-operatively or with simple Burr hole evacuation respectively.

Specific traumatic brain injuries (TBI): Focal injuries: Epidural hematomas (EDH) Continued

i. Clinical presentation: Depends on the source and rapidity of bleeding (1) Palpate temple for boggy hematoma indicating trauma to the vessels that spread out to the temporalis muscle with possible underlying temporal skull fx. (2) If brain shift: Ipsilateral dilated pupil that becomes fixed due to pressure on CN III. Loss of eye motor abduction on the affected side due to pressure on CN VI. Localized contralateral extremity weakness. Very important sign. Look for pronator drift. (3) Change in consciousness: GCS drops by 2-3 points. May have a lucid interval (9%-33%) between injury and loss of consciousness. (4) Headache of increasing severity (5) Possible seizures, vomiting (6) Beware posterior fossa epidural hematoma: 5-10% of all EDHs, but carry 26% of the mortality rate. (a) Occipital skull fracture (b) Cranial nerve dysfunction (c) Cerebellar signs (d) Abnormal extraocular movements (e) Motor dysfunction (f) Nausea/vomiting (g) ↓ VS and respiratory arrest (h) May be delayed in appearance j. Diagnostic radiography: Lens-shaped on CT. HYPERdense. Brain is 70% water, cannot be compressed so shift compresses ventricles. k. Emergency interventions: Surgical evacuation if S&S of brain shift or >30mL volume. Not all require surgery; may resolve in a couple of days if small.

GCS missing information

i. There are various different strategies for dealing with information that is missing because of factors interfering with assessment. This is the recommended approach from the founders' www.GasgowComaScale.org website being adopted by ATLS, NTDS, TQIP and other standard trauma data collection groups: (1) "Assess, communicate and make decisions using the remaining components. Although guidelines are often expressed in terms of a total GCS 'score,' the trend in whichever of the components (eye, motor or verbal) can be assessed is still valuable. (2) Do not use number '1' (None) to record missing component; use 'NT' (Not testable). (3) Do not report a total score when a component is Not Testable because the score will be low and this could be confusing to medical colleagues. This may also imply that the patient is more unwell than they actually are. (4) It is possible using statistical methods to estimate a missing component from the findings in the other components. This is probably more relevant to research than clinical practice." j. A new score integrating pupillary reactivity and GCS is evolving for use in clinical prognostication and will be incorporated into the 2019 TQIP admission database with more research to follow. k. Young children: Champion et al described a better method to describe the verbal responses of children younger than three years: (1) 5: Appropriate for age (2) 4: Consolable cries (3) 3: Persistent irritation (4) 2: Restlessness and agitation (5) 1: No verbal response

Mild brain injury (MBI)

or concussion: GCS 13-15 (ACS, 2018). American Academy of Neurology defines concussion as "trauma induced alteration in mental status that may or may not involve a loss of consciousness". Only 3% should deteriorate. Symptoms include temporary headaches, memory disturbance, dizziness, irritability, fatigue, mild mental slowing with decreased concentration and attention span, impaired perception or mood, sleep disturbances, sensitivity to noise or light, and balance problems. They almost always improve over one to three months. Infants and young children may have observed signs of irritability, lethargy or vomiting following MBI.


Ensembles d'études connexes

Diet therapy for clinical nutrition, modules 1-6 ( Understanding normal and clinical nutrition, tenth edition)

View Set

OSHA: Using Power tools - Part 2

View Set

Chap 11 sb Capacity and Legality

View Set

spanish vocab: mirar (to look at)

View Set

Drawing Trend Lines Assignment and Quiz 100%

View Set

PT 3: Oncology: Prostate Cancer (EXAM 10)

View Set